Anda di halaman 1dari 33

1.

Paciente varn de 52 aos, del que desconocemos


los antecedentes personales, que a la llegada al ser-
vicio de urgencias presenta bajo nivel de concien-
cia, con escasa reaccin a los estmulos externos,
pero con respuesta conservada al dolor. Los miem-
bros del 061 nos reeren que lo han encontrado en
la va pblica, tras un cuadro que, segn los tes-
tigos, pareca una crisis convulsiva, tras el que el
paciente presentaba cierta confusin y refera sen-
sacin de entumecimiento en la cara y en ambos
miembros superiores. Entre la batera de pruebas
que hemos solicitado en urgencias tenemos una
gasometra arterial: PH 7,53; PaO
2
98, PaCO
2
53,
HCO
3
42. Ante el cuadro clnico y esta alteracin
analtica, pensaremos que el paciente presenta:
a) Acidosis metablica.
b) Alcalosis mixta.
c) Alcalosis metablica.
d) Alcalosis respiratoria.
e) Acidosis respiratoria.
2. Mara es una mujer de 25 aos, que al comenzar
el invierno debuta clnicamente con un cuadro de
tos, con escasa produccin de moco, y sibilancias.
Al estudiar a la paciente, en las pruebas funciona-
les se detecta un patrn obstructivo, con mejora
tras la prueba broncodilatadora. En el estudio de
sangre perifrica, as como en del esputo, se detec-
ta eosinolia. Seale la armacin errnea acerca
del tratamiento del asma:
a) Una opcin adecuada para el asma leve persisten-
te podra ser budesonida pautada ms salbutamol
a demanda.
b) En el asma leve intermitente, basta el empleo de
beta-2-agonistas en el momento de las crisis.
c) Para el asma por ejercicio, puede ser til el uso de
cromoglicato antes del mismo.
d) A diferencia del EPOC, los anticolinrgicos tie-
nen un papel ms relevante en el tratamiento del
asma que los corticoides inhalados.
e) Son efectos frecuentes de los corticoides inhala-
dos la disfona, la tos y la irritacin farngea.
SIMULACRO 1
3. Un paciente de 62 aos, exfumador reciente, con-
sulta por disnea de varios meses de evolucin y tos
no productiva. En ambas manos, existen acropa-
quias muy evidentes. En la auscultacin, existen
crepitantes teleinspiratorios y, en la Rx trax se
aprecia un patrn reticular. La pletismografa ob-
jetiva una prdida de volumen y la TC conrma
los hallazgos radiogrcos. Si se realiza un lavado
broncoalveolar, sera esperable:
a) Predominio de linfocitos CD4+sobre los CD8+.
b) Predominio de los linfocitos CD8+sobre los CD4+.
c) Aumento de los neutrlos.
d) Aumento de los eosinlos por encima del 50%.
e) Incremento de las clulas de Langerhans.
4. Un paciente de 60 aos, obeso y diabtico, acude
a consulta con su esposa porque, segn ella, por
la noche emite intensos ronquidos y hace pausas
apneicas mientras duerme, con movimientos to-
rcicos durante las mismas. Durante el da, oca-
sionalmente acusa cierta somnolencia, aunque
no llega a quedarse dormido. Se le realiza una
polisomnografa y el registro muestra un ndice
de apnea-hipopnea de 18/hora, con apneas fun-
damentalmente obstructivas. Seale la respues-
ta correcta:
a) Puede establecerse el diagnstico de sndrome de
apnea obstructiva del sueo.
b) Se trata de un roncador simple y no precisa trata-
miento.
c) Est indicado comenzar tratamiento con CPAP.
d) Se trata de un sndrome de apneas centrales, se-
cundarias a una insuciencia cardaca todava no
diagnosticada.
e) El cuadro es sugestivo de hipoventilacin alveolar
primaria.
5. Seale la armacin incorrecta en relacin con la
clnica del asma:
booksmedicos.org
-2-
SI MULACRO 1
a) Radiolgicamente, lo habitual es encontrar una
Rx trax normal, salvo en las crisis graves.
b) El neumotrax y el neumomediastino pueden ser
complicaciones de las crisis asmticas.
c) El dato auscultatorio ms tpico son las sibilancias
inspiratorias.
d) La PaCO
2
suele estar disminuida durante las crisis.
e) En crisis asmticas graves puede aparecer pulso
paradjico.
6. Un ADVP con ebre de dos meses de evolucin,
poliadenopatas, candidiasis oral y deterioro pro-
gresivo es diagnosticado de infeccin por VIH y
tuberculosis por una tincin de Ziehl-Neelsen de
un aspirado ganglionar. La Rx trax es normal.
Mantoux negativo. Cul es FALSA en relacin a
la tuberculosis (TBC)?
a) La TBC se produce en ms de los individuos in-
fectados previamente (Mantoux +) con infeccin
por VIH si no reciben quimioprolaxis con iso-
niacida.
b) La TBC generalmente precede a otras infecciones
oportunistas denitorias de SIDA.
c) La prueba de la tuberculina tiene un elevado n-
mero de falsos negativos en estos pacientes.
d) El tratamiento antituberculoso debe prolongarse
ms de 12 meses en el paciente VIH.
e) La TBC del paciente VIH suele ser por la reacti-
vacin de un foco tuberculoso antiguo.
7. Mujer de 55 aos, fumadora activa, que consulta
por dolor de hombro izquierdo irradiado a brazo
de varias semanas de duracin. En la exploracin
fsica destaca ptosis y miosis del ojo izquierdo, por
lo que usted solicita una Rx trax, encontrndose
una condensacin en vrtice pulmonar izquierdo
e imgenes sugestivas de erosin en las primeras
costillas. Qu tipo histolgico es el ms probable-
mente implicado?
a) Osteosarcoma.
b) Carcinoma anaplsico de clulas grandes.
c) Carcinoma microctico.
d) Adenocarcinoma.
e) Carcinoma epidermoide.
8. Varn de 68 aos con antecedentes de cncer
de prstata, intervenido hace 10 das, sin otros
problemas de salud. Acude a Urgencias por dis-
nea sbita. La exploracin revela una presin
venosa a 5 cm sobre la horizontal, 110 lpm, 25
rpm. Afebril. Resto sin alteraciones. Rx trax:
pequeas atelectasias laminares bibasales de
difcil apreciacin, por lo dems normal. ECG:
ritmo sinusal a 110 lpm, con ondas T invertidas
en precordiales derechas. Teniendo en cuenta el
diagnstico ms probable seale la respuesta co-
rrecta:
a) Debera conrmarse con arteriografa.
b) Antes de iniciar ninguna maniobra teraputica,
habra que realizar una gammagrafa ventilacin-
perfusin o un TC helicoidal.
c) El cncer de prstata no es un factor predisponen-
te.
d) La gravedad del cuadro y la alta probabilidad cl-
nica de padecerlo aconsejan tratar directamente
con anticoagulantes.
e) La Rx trax es sugestiva de neumona nosocomial.
9. Paciente de 40 aos, con cuadro de dos das de
evolucin de tos y ebre de 39. En la radiografa
de trax se observa la existencia de un derrame
pleural que ocupa la mitad del hemitrax izquier-
do. En este mismo hemitrax se ausculta un soplo
tubrico. Los hallazgos analticos de la toracocen-
tesis son: pH 7,0, LDH 1500 u/l, protenas 3,9. La
toracocentesis fue difcil, por lo que se realiza una
ecografa, que muestra que el derrame est par-
cialmente tabicado Cul ser la conducta ms
apropiada a seguir?
a) Iniciar tratamiento antibitico emprico y esperar
los resultados del cultivo del lquido pleural para
colocar un tubo de drenaje torcico.
b) Colocar un tubo de drenaje torcico e iniciar trata-
miento antibitico emprico.
c) Iniciar tratamiento antibitico emprico y repe-
tir la toracentesis al cabo de 24 horas para valo-
rar la actitud a seguir en funcin de la evolucin
de los parmetros bioqumicos del lquido pleu-
ral.
d) Colocar un drenaje pleural, instilar brinolticos
intrapleurales e iniciar tratamiento antibitico em-
prico.
e) Efectuar una broncoscopia para toma de mues-
tras microbiolgicas, colocar un tubo de dre-
naje torcico e iniciar tratamiento antibitico
emprico.
10. Varn de 58 aos, fumador de 20 cigarrillos dia-
rios, padece un cuadro de anorexia, astenia y pr-
dida de peso. Se realiza un estudio radiolgico
donde se encuentra un ndulo pulmonar solitario
de 3 cm, parahiliar, en pulmn derecho. No se dis-
pone de radiografas previas. El ndulo no est
calcicado ni cavitado. La broncoscopia encuen-
booksmedicos.org
-3-
SI MULACRO 1
tra una masa endobronquial y la biopsia describe
clulas en grano de avena (oat cell). Entre los si-
guientes pptidos hormonales, cul es el que NO
es segregado por este tipo de tumores:
a) ACTH.
b) PTH.
c) Calcitonina.
d) Pptido intestinal vasoactivo.
e) Gastrina.
11. Un paciente previamente sano es ingresado por
una sepsis de origen abdominal. A las 24 horas del
ingreso comienza con disnea y taquipnea de cre-
ciente intensidad. Una radiografa de trax mues-
tra inltrados alveolares bilaterales, de predomi-
nio en bases, que pasadas unas horas se extienden
por los dos pulmones. La PaO
2
es de 52 mmHg, a
pesar de la oxigenoterapia en altas concentracio-
nes. Cul es su diagnstico?
a) Tromboembolismo pulmonar secundario a la in-
movilizacin.
b) Absceso en hemiabdomen superior.
c) Sndrome de distrs respiratorio del adulto.
d) Hemorragia alveolar.
e) Derrame por pancreatitis aguda necrohemorr-
gica.
12. Una paciente de 40 aos consulta por un ndulo
tiroideo indoloro, de 3-4 cm de dimetro y reciente
aparicin. Clnicamente, slo reere tendencia a
la diarrea y enrojecimiento facial sbito en algu-
na ocasin. La calcitonina plasmtica est elevada
y en la PAAF se ha documentado la presencia de
sustancia amiloide. El diagnstico ms probable
sera:
a) Carcinoma papilar tiroideo.
b) Carcinoma folicular tiroideo.
c) Carcinoma medular de tiroides.
d) Adenoma folicular.
e) Linfoma folicular.
13. Una mujer de 34 aos consulta por dolor en re-
gin anterior del cuello de aproximadamente
un mes de evolucin, que ocasionalmente tam-
bin se irradia a odo. Ha tenido ebre de 38,4
C, prdida de peso sin reduccin de la ingesta,
temblor distal y diarrea. Desde el punto de vista
analtico, destaca una elevacin de la VSG y la
TSH abolida. La T4 libre est elevada. Capta-
cin tiroidea deprimida. Seale cul de las si-
guientes armaciones es FALSA en relacin con
el cuadro que padece:
a) El propranolol podra colaborar al buen control de
los sntomas.
b) En casos graves, puede recurrirse a los corticoides.
c) El tratamiento de eleccin son los antiinamato-
rios no esteroideos.
d) Como en la enfermedad de Graves, la TSH est
suprimida.
e) Una buena opcin teraputica seran los antitiroi-
deos.
14. En una paciente de 55 aos se encuentra una cifra
de calcio de 11,2 mg/dl (normal menos de 10,5 mg/
dl) en un anlisis que se realiza por otro motivo.
La determinacin de PTH solicitada en el estudio
de la hipercalcemia demostr una cifra de 127 pg/
ml (normal hasta 45 pg/ml). La densidad sea, el
aclaramiento de creatinina y la calciuria son nor-
males. Sospecha un hiperparatiroidismo prima-
rio. Cul de las siguientes lesiones paratiroideas
producen este cuadro con mayor frecuencia?
a) Carcinoma.
b) Adenoma solitario.
c) Adenoma mltiple.
d) Hiperplasia de clulas principales.
e) Hiperplasia de clulas oxlas.
15. Un varn de 53 aos, estudiado por hipotensin,
presenta como datos analticos ms importan-
tes: T4 srica 2,3 ug/dl, T3 92 ng/dl, TSH 75 mU/
ml, anticuerpos antimicrosomales positivos al
1/600.000, sodio 126 mEq/l y potasio 5,5 mEq/l.
Con los datos disponibles en este momento, seale
qu actitud sera la ms correcta:
a) Diagnosticar hipotiroidismo secundario y comen-
zar estudio hiposario.
b) Diagnosticar hipotiroidismo primario y comenzar
tratamiento con levotiroxina.
c) Sospechar sndrome eutiroideo de la T3 baja
(eutiroideo enfermo).
d) Determinar la respuesta de cortisol a ACTH.
e) Sospechar secrecin inadecuada de ADH y buscar
carcinoma oculto.
16. Juan es un paciente de 35 aos de edad que acu-
de a nuestra consulta solicitando ser estudiado, a
pesar de no presentar ningn tipo de sintomato-
booksmedicos.org
-4-
SI MULACRO 1
loga, ya que a su padre le acaban de diagnosticar
un sndrome MEN tipo I, y le han comunicado el
carcter hereditario de la enfermedad. Adems de
explicarle que esta patologa se hereda con carc-
ter autosmico dominante, estando localizada la
alteracin a nivel del cromosoma 11, le explicare-
mos en que consiste dicha patologa y las pruebas
que le vamos a solicitar para descartar la afecta-
cin endocrina mltiple. Entre las asociaciones
que caracterizan al MEN tipo 1 no se encuentra:
a) Hiperparatiroidismo.
b) Gastrinoma.
c) Carcinoma medular de tiroides.
d) Tumores hiposarios.
e) Adenoma suprarrenal no funcionante.
17. Doa Francisca es una mujer de 62 aos de edad,
que mide 1,59 m y pesa 85 Kg. Hace 6 meses, en
un estudio analtico de rutina, se detect una hi-
perglucemia, que se conrm posteriormente en
nuevos estudios analticos. Se inici tratamiento
diettico que se ha demostrado insuciente para
el control de sus cifras de glucemia, motivo por el
cul se plantea en el momento actual iniciar tra-
tamiento con hipoglicemiantes orales. En relacin
con los hipoglicemiantes orales es cierto que:
a) La repaglinida no aumenta la secrecin de insulina.
b) La acarbosa acta sobre los receptores nucleares
PPAR.
c) Las tiazolidindionas disminuyen la resistencia in-
sulnica.
d) Est contraindicada la combinacin de dos hipo-
glucemiantes.
e) La metformina produce hipoglucemias con ms
frecuencia que las sulfonilureas.
18. En relacin al aumento de la prevalencia de la
obesidad y la inactividad fsica en la poblacin,
se ha incrementado la frecuencia de un trastorno
denominado sndrome metablico, sndrome X o
sndrome de Raven. Seale la respuesta INCO-
RRECTA en relacin con el mismo:
a) El sustrato siopatolgico es la resistencia insulnica.
b) El incremento del colesterol-LDL no se considera
como uno de sus criterios.
c) Uno de sus criterios diagnsticos es la hipertrigli-
ceridemia.
d) La hipertensin arterial entrara en el concepto de
este sndrome.
e) El descenso del colesterol-HDL no forma parte
del sndrome metablico.
19. Cul de las siguientes armaciones sobre las ca-
ractersticas clnicas de la artritis reumatoide es
INCORRECTA?
a) La afectacin de la columna cervical es ms fre-
cuente que la lumbar.
b) La afectacin de las articulaciones interfalngicas
proximales es ms frecuente que las distales.
c) La manifestacin pulmonar ms frecuente es la
pleuritis, aunque suele ser asintomtica.
d) La evidencia radiolgica de aumento de partes
blandas periarticular forma parte de sus criterios
diagnsticos.
e) La artritis reumatoide del adulto se asocia con un
aumento de la frecuencia de ojo seco, episcleritis
e incluso escleritis.
20. Paciente varn de 55 aos de edad, con antece-
dente personal de asma intrnseca de larga evo-
lucin (diagnosticado a los 35 aos de edad), en
tratamiento con corticoides, que de forma re-
ciente ha comenzado con clnica de frecuentes
episodios de broncoespasmo, que han precisado
atencin mdica repetida en el servicio de Ur-
gencias. Entre las pruebas realizadas, tenemos
una Rx de trax en la que se aprecia una cardio-
megalia, motivo por el que le remiten a nuestra
consulta. Realizamos un ecocardio transesofgi-
co que es informado como: ventrculo izquierdo
sin engrosamiento de la pared y funcin sistlica
dentro de la normalidad, ligera insuciencia mi-
tral, con vlvula calcicada, y derrame pericr-
dico, con signos de taponamiento cardaco. Se
realiz una pericardiocentesis, siendo el lqui-
do obtenido de caractersticas de exudado, con
predominio de PMN y sin signos de malignidad.
Los cultivos microbiolgicos tambin resultaron
negativos, del mismo modo que los marcadores
tumorales, los anticuerpos antinucleares y el
factor reumatoide. Tras estas y algunas pruebas
ms se dio de alta al paciente con el diagnstico
de derrame pericrdico idioptico. Al cabo de
un mes ingresa en el servicio de urgencias con
dolor torcico y nueva crisis de broncoespasmo,
a la que se ha sumado desde hace dos das, una
sensacin de acorchamiento y prdida de fuerza
en la regin ms distal de la pierna izquierda.
En el estudio Rx presenta inltrados pulmona-
res. Al descartarse, mediante broncoscopia y la-
vado alveolar, la patologa infecciosa y tumoral,
el diagnstico de sospecha se dirige hacia el sn-
drome de Churg-Strauss. En pacientes con an-
getis y granulomatosis alrgica (enfermedad de
Churg-Strauss), el hallazgo de laboratorio ms
caracterstico, por su frecuencia y utilidad para
el diagnstico diferencial, sera:
booksmedicos.org
-5-
SI MULACRO 1
a) La trombocitosis.
b) La elevacin de la VSG.
c) La eosinolia en sangre perifrica.
d) La presencia de neutrlos en orina.
e) La anemia microangioptica.
21. Paciente varn de 27 aos de edad, sin ante-
cedentes personales de inters, que acude a la
consulta reriendo dolor lumbar de 6 meses de
evolucin. Reere que casi todos los das el do-
lor le despierta a lo largo de la noche y que al
levantarme de la cama siento como si no pudiera
moverme y que al ir, poco a poco, movindome el
dolor mejora. Reere que el dolor mejora por
la actividad y empeora con el reposo. A la explo-
racin presenta maniobras de Lasegue y Bra-
gard positivas a 15 para ambos lados, as como
dolor a la palpacin en la regin lumbar baja,
sobre las apsis espinosas y la musculatura pa-
ravertebral, y en la regin de las articulaciones
sacroiliacas. La movilidad de la columna lumbar
est discretamente disminuida. Se realiza estu-
dio analtico con valores de PCR de 2,1 (normal
hasta 0,5) y VSG de 90 mm/h (normal hasta 20
mm/h). En el estudio radiolgico simple aprecia-
mos signos de sacroileitis, as como sindesmo-
tos en la regin lumbar. Con todo esto se llega
al diagnstico de espondilitis anquilosante. La
manifestacin extraarticular ms frecuente de la
espondilitis anquilosante es:
a) La uvetis anterior aguda.
b) Insuciencia artica.
c) Prostatitis crnica.
d) Fibrosis en lbulos superiores.
e) Iletis terminal.
22. En una paciente de 70 aos con arteritis de clulas
gigantes. Cul de los siguientes armaciones con-
siderara FALSA?
a) La leucocitosis no es frecuente ni es sugestiva de
esta enfermedad.
b) El parmetro analtico que mejor se correlaciona
con la evolucin clnica es la anemia.
c) El sntoma ms habitual es la cefalea.
d) La claudicacin mandibular es un dato tpico.
e) Una biopsia normal no excluye denitivamente el
diagnstico.
23. Seale, entre las siguientes opciones, la que mejor
encaje con el diagnstico de panarteritis nodosa
clsica (PAN):
a) Asma, eosinolia e inltrados pulmonares no ca-
vitados.
b) Varn de 40 aos con artromialgias, HTA y dolor
abdominal.
c) Sinusitis, epistaxis, ndulos pulmonares cavita-
dos y dao renal.
d) Anciano con claudicacin mandibular, cefalea y
afectacin oftlmica.
e) Varn joven con hipertensin intracraneal, ebitis
y erupcin acneiforme.
24. Respecto a la relacin entre gota y nefrolitiasis, es
CIERTO que:
a) Todos los individuos con litiasis rica terminan
desarrollando otras manifestaciones de gota.
b) El riesgo de litiasis rica es muy alto cuando la
uricosuria est entre 400 y 600 mg diarios.
c) Los gotosos sufren una mayor incidencia de litia-
sis clcica que la poblacin general.
d) La mayora de los pacientes con litiasis clcica
son, adems, gotosos.
e) Cuando un individuo gotoso presenta litiasis ri-
ca, puede asegurarse que ya sufre un deterioro sig-
nicativo de la funcin renal.
25. Paciente con antecedentes de artritis reumatoi-
de, en tratamiento con metotrexato. En la ltima
revisin estaba bien controlada, con hemograma
y bioqumica normales. Sin embargo, hoy acu-
de al Servicio de Urgencias por ebre de 38,7
C, dolor y tumefaccin de 12 horas de evolucin
en la rodilla izquierda. En la exploracin, usted
objetiva dolor a la palpacin, eritema y aumento
de calor en dicha zona. Cul sera la actitud
ms adecuada?
a) Artrocentesis, examen del lquido sinovial con
microscopio de luz polarizada y test de Gram ur-
gente.
b) Instaurar tratamiento esteroideo en altas dosis.
c) Iniximab i.v.
d) Inltracin intraarticular de esteroides.
e) Antiinamatorios no esteroideos, protector gstri-
co y derivar a Reumatologa.
26. Un varn de 85 aos, obeso, consulta por un do-
lor inguinal de varios meses de evolucin, que se
desencadena con la marcha y mejora en reposo,
lo que le obliga a cojear. Tambin le produce di-
cultad para sentarse o levantarse de una silla.
No tiene otros sntomas, salvo los descritos. La
exploracin articular es dolorosa y evidencia una
booksmedicos.org
-6-
SI MULACRO 1
disminucin de la rotacin interna articular. No se
aprecian signos inamatorios. Teniendo en cuenta
el diagnstico ms probable, seale la armacin
INCORRECTA:
a) El tratamiento inicial es sintomtico y podra ser
paracetamol.
b) La prdida de peso podra ser beneciosa.
c) Radiolgicamente sera lgico encontrar pinza-
miento del espacio articular y esclerosis subcon-
dral.
d) Lo ms probable es que el lquido sinovial sea in-
amatorio durante las pocas de ms dolor.
e) En ms de la mitad de los casos hay factores pre-
disponentes, siendo uno de ellos la luxacin con-
gnita de cadera.
27. Paciente de 31 aos que acude al Servicio de Ur-
gencias tras un accidente en bicicleta, donde cay
sobre el brazo derecho. La exploracin clnica re-
vela dolor, movilidad anormal e impotencia fun-
cional en la zona media del brazo. Los pulsos dis-
tales humeral y radial estn presentes. No puede
realizar la extensin activa de la mueca y dedos.
Existe una hipoestesia en la zona dorsal del pri-
mer dedo y del primer espacio interdigital. Las
radiografas muestran una fractura espiroidea del
tercio medio del hmero. Cul es la actitud ms
correcta respecto a la lesin asociada a esta frac-
tura?
a) Requiere ciruga inmediata y exploracin quirr-
gica del nervio radial.
b) Precisa TC cerebral y RMN cervical.
c) El tratamiento es conservador, con ejercicios de
rehabilitacin posteriores.
d) Debemos informar al paciente de que lo ms pro-
bable es la irreversibilidad de la prdida de fun-
cin.
e) El cuadro es sugerente de sndrome comparti-
mental y es imperativa una fasciotoma urgen-
te.
28. Un chico de 21 aos, durante un partido de ft-
bol, da una fuerte patada al vaco cuando inten-
taba disparar a portera. Como consecuencia de
un mecanismo de hiperextensin de la rodilla y
de forma inmediata, siente un chasquido y su-
fre un intenso dolor en la rodilla derecha. En la
exploracin, en slo 30 minutos, se evidencia un
derrame articular importante, que se conrma
posteriormente al realizar una artrocentesis, ob-
teniendo 60 cc de lquido con caractersticas de
hemartros. A pesar del dolor y de una pobre co-
laboracin por parte del paciente, se realiza el
test de Lachman con resultado positivo. Seale
el diagnstico ms probable:
a) Lesin del ligamento cruzado anterior.
b) Lesin del ligamento cruzado posterior.
c) Lesin del menisco medial.
d) Lesin de la cpsula articular.
e) Lesin del ligamento colateral medial.
29. Trasladan al Servicio de Urgencias, desde un cen-
tro de salud situado a 100 km del hospital, a un
paciente varn de 35 aos de edad, que ha sufrido
una fractura abierta de tobillo izquierdo tras una
cada mientras practicaba descenso de barrancos
en la montaa. El paciente es trasladado con el to-
billo inmovilizado con una frula posterior y, se-
gn consta en el informe del traslado, ya ha sido
tratado con antibioterapia (protocolo de fracturas
abiertas), prolaxis antitetnica y lavado inicial
de la herida con suero abundante. A continuacin,
la actitud inicial ms correcta sera, entre las si-
guientes:
a) Osteosntesis con placa y tornillos.
b) Fijacin externa.
c) Traccin esqueltica.
d) Traccin cutnea.
e) Fijacin interna con agujas de Kirschner.
30. Acude a la urgencia una mujer de 60 aos, con im-
portante dolor en el brazo derecho, que ha apoya-
do contra el suelo despus de tropezar en la calle y
caer hacia delante. En la exploracin se evidencia
una deformidad en dorso de tenedor en su mueca
derecha. Se realizan unas radiografas que con-
rman la existencia de una fractura que afecta al
radio distal. Seale la complicacin ms frecuente
de esta fractura:
a) Pseudoartrosis.
b) Ruptura del tendn del extensor largo del pul-
gar.
c) Ruptura del tendn del exor largo del pulgar.
d) Ruptura del tendn del palmar mayor.
e) Compresin del nervio mediano a nivel del tnel
del carpo.
31. Un paciente de 18 aos, con antecedentes per-
sonales de enfermedad de Osgood-Schlatter en
la rodilla derecha y secuelas de enfermedad
de Perthes en la cadera izquierda, consulta
por una lesin deportiva de la rodilla, que se
booksmedicos.org
-7-
SI MULACRO 1
produjo hace un mes mientras jugaba al ba-
loncesto. Refiere una historia que sugiere un
traumatismo por torsin, tras la que el pa-
ciente fue atendido en el servicio de Urgencias
presentando un cuadro de intenso dolor, con
impotencia funcional, y bloqueo de la rodilla
en flexin de 30, que fue resuelto mediante
movilizacin bajo anestesia local intraarti-
cular. A las pocas horas la rodilla presentaba
importante tumefaccin, con signo de peloteo
positivo, y dificultad para la deambulacin
por dolor. En los siguientes diez das el cua-
dro clnico cedi por completo con tratamiento
antiinflamatorio. Pero al ponerse en cuclillas
reapareci el cuadro clnico, que posterior-
mente ha continuado con presencia ocasional
de bloqueos, claudicacin y derrame sinovial.
Esto nos hace sospechar:
a) Lesin de ligamento cruzado anterior.
b) Lesin meniscal.
c) Lesin del ligamento cruzado posterior.
d) Condromalacia rotuliana.
e) Osteocondritis de los cndilos femorales.
32. Un hombre de 21 aos sufre un accidente de
trfico, saliendo despedido del vehculo a tra-
vs del parabrisas al no llevar puesto el cin-
turn de seguridad. Ingresa en la Urgencia
de Traumatologa con un cuadro de prdida
progresiva de fuerza en miembros inferiores,
sin que a la exploracin fsica presente defor-
midades externas en los MMII. Los reflejos
osteotendinosos estn abolidos. El paciente
refiere de forma exclusiva dolor en la regin
lumbar. El estudio con radiografas simples
muestran una fractura por compresin de L1,
con desplazamiento del muro posterior y acu-
amiento anterior de un 50%. El resto de las
radiografas simples descartan la existencia
de lesiones seas agudas en otras localizacio-
nes. Qu prueba diagnstica de las siguientes
indicara para valorar la ocupacin del canal
raqudeo?
a) Una termografa.
b) Una densitometra sea.
c) Una TAC vertebral centrada en regin dorso-
lumbar.
d) Una gammagrafa sea.
e) Una tomografa cervical.
33. Un paciente de 30 aos ingres hace 3 das por
una fractura subtrocantrea de fmur izquierdo,
fractura de rtula derecha y fractura diasaria
conminuta de tibia izquierda, que se inmoviliza-
ron provisionalmente, en espera de ciruga de os-
teosntesis. Sbitamente, comienza con estupor,
disnea, taquipnea y petequias conjuntivales, que
despus se extienden por el hemitrax superior.
Seale la respuesta INCORRECTA en relacin
con esta entidad:
a) El diagnstico ms probable es la embolia gra-
sa.
b) El cuadro clnico que sospechamos se asocia
fundamentalmente a la jacin tarda de las
fracturas y con cierta frecuencia al shock hipo-
volmico.
c) Es tpico de esta entidad un intervalo lcido de
2-3 das desde el momento del traumatismo.
d) El tratamiento consistir en oxigenoterapia, cor-
ticoides en altas dosis y la estabilizacin de las
fracturas.
e) La imagen radiolgica ms probable es una Rx de
trax normal.
34. Un paciente de 50 aos acude a Urgencias con un
intenso dolor en el ojo izquierdo, que le sobrevino
de forma brusca mientras dorma. En la explora-
cin, el ojo afecto est enrojecido, con discreto ede-
ma corneal, y la pupila se encuentra en midriasis
media, con pobre respuesta a la luz. Teniendo en
cuenta el diagnstico ms probable, qu opcin
le parece MENOS adecuada, entre las siguientes
propuestas?
a) Si el paciente es hipermtrope, probablemente
haya contribuido como factor predisponente.
b) Podra ayudarle el uso de manitol o acetazolamida.
c) Sera aconsejable el uso de midriticos para rom-
per el bloqueo pupilar.
d) Los corticoides tpicos seran de utilidad para dis-
minuir el edema y la reaccin inamatoria.
e) Es probable que este paciente reera ver halos al-
rededor de las luces.
35. Mujer de 36 aos que acude a consulta por dolor
agudo y enrojecimiento del ojo derecho, de 2 das
de evolucin. La exploracin del ojo nos revela los
siguientes datos: discreta disminucin de la agude-
za visual, presin intraocular de 17 mmHg, pupila
en miosis, presencia de clulas en humor acuoso,
opacidad cristaliniana moderada y presencia de
una sinequia iris-cristalino. Seale la respuesta
correcta sobre el caso descrito:
a) Se trata de un caso de neuritis ptica y debemos
descartar una esclerosis mltiple asociada.
booksmedicos.org
-8-
SI MULACRO 1
b) Ser benecioso instaurar tratamiento tpico con
esteroides y midriticos.
c) Se trata de una uvetis toxoplsmica, que es la
causa ms frecuente de uvetis posterior.
d) Sera til comenzar tratamiento con manitol intra-
venoso, colirios miticos y corticoides tpicos.
e) Aunque lo ms frecuente es que sea idioptica, la
enfermedad ms relacionada con esta entidad es la
sarcoidosis.
36. Como muchas otras enfermedades sistmicas, la
afeccin por el virus de la inmunodeciencia hu-
mana (VIH) se ha relacionado con manifestacio-
nes oculares: se han descrito alteraciones intrao-
culares, tanto en el segmento anterior como en el
segmento posterior, alteraciones neurooftalmol-
gicas y alteraciones a nivel de la rbita. La mani-
festacin oftalmolgica ms frecuente del VIH es:
a) Retinitis por CMV.
b) Retinopata SIDA.
c) Necrosis retiniana aguda.
d) Queratouvetis por herpes zster.
e) Coriorretinitis por Toxoplasma.
37. Una paciente de 68 aos, diabtica tipo 2 en tra-
tamiento con insulina desde hace 4 aos, consulta
por una prdida repentina de visin en el ojo de-
recho. La agudeza visual es de un 20%, cuando el
mes pasado (ltima revisin) era de casi un 100%.
No tiene dolor. El segmento anterior del ojo es nor-
mal. Teniendo en cuenta la causa ms probable de
su prdida visual, seale la actitud inicial:
a) Colirios miticos e hipotensores oculares.
b) Observacin.
c) Vitrectoma.
d) Triamcinolona intravtrea.
e) Antibioterapia intravtrea.
38. Una paciente acude al servicio de urgencias con
ebre y mal estado general, presentando en la
exploracin fsica un empastamiento doloroso
del suelo de la boca, desde hace tres das, segn
reere la paciente. Existe franco abombamiento
de la regin submentoniana. La paciente reere
asimismo que adems de las crecientes diculta-
des para tragar los alimentos experimentadas los
das previos, en las ltimas horas ha comenzado
a experimentar algunas dicultades para respi-
rar. En relacin a la enfermedad que sospecha,
es cierto que:
a) Si presenta agranulocitosis, se hablara de angina
de Schultze.
b) Se trata de una angina de Plaut-Vincent, produci-
da por anaerobios y espiroquetas.
c) En el tratamiento mdico deben incluirse antimi-
cticos.
d) El diagnstico ms probable es el de herpangina,
producida por el virus Coxsackie A.
e) Probablemente est en relacin con mal estado
dentario.
39. Una mujer muy fumadora y habladora presenta
episodios de disfona. Durante las ltimas sema-
nas, esta disfona es cada vez ms intensa. La la-
ringoscopia muestra una neoformacin gelatinosa
que afecta a ambas cuerdas vocales, cuya movi-
lidad no est afectada. No existen adenopatas
cervicales. Teniendo en cuenta el diagnstico ms
probable, el tratamiento ser:
a) Cordectoma bilateral.
b) Antiinamatorios no esteroideos +mucolticos.
c) Decorticacin bilateral.
d) Vaporizacin con lser de CO
2
.
e) Radioterapia.
40. El absceso periamigdalino es la complicacin ms
frecuente de la amigdalitis aguda. Respecto al mis-
mo, seale la armacin correcta:
a) La vula se desplaza al lado sano.
b) Produce tortcolis por afectacin de la fascia del
esternocleidomastoideo.
c) Es la causa ms frecuente del sndrome de Lemirre.
d) El tratamiento de eleccin es la clindamicina.
e) El dolor se hace unilateral.
41. Respecto a las otitis externas, seale la asociacin
incorrecta:
a) Otitis externa maligna - Virus herpes zster.
b) Otitis externa bacteriana - Otalgia.
c) P. aeruginosa - Otitis externa difusa.
d) S. aureus - Otitis externa circunscrita (fornculo
del CAE).
e) Otomicosis - Prurito.
42. Respecto al sndrome de Menire, una de las si-
guientes armaciones no es correcta. Selela:
booksmedicos.org
-9-
SI MULACRO 1
a) Produce una hipoacusia neurosensorial, uctuante
y con mayor afectacin de los tonos graves.
b) Dado que la hipoacusia es de origen coclear, es
lgico encontrar reclutamiento.
c) La mayora de los pacientes tienen afectacin bi-
lateral.
d) En la patogenia, se implica una sobreproduccin
de endolinfa (hydrops endolinftico).
e) Durante las crisis, aparece espontneamente un
vrtigo muy intenso, de caractersticas perifricas.
43. En la esquizofrenia, Andreasen sistematiz los sn-
tomas en positivos y negativos, y esta clasicacin
sigue vigente hasta nuestros das. Seale cul de
los siguientes es un sntoma negativo:
a) Delirio de inuencia.
b) Pseudoalucinaciones auditivas.
c) Ensalada de palabras.
d) Frialdad emocional.
e) Tangencialidad del discurso.
44. Cul de los siguientes enunciados respecto a la
depresin mayor o episodio depresivo mayor es
INCORRECTO?
a) Ante la sospecha de episodio depresivo mayor, es
importante valorar el riesgo suicida, por lo que es
importante preguntar por ideas o proyectos de este
tipo.
b) Entre las alteraciones del sueo, que pueden ser
variables, es tpica la presencia de insomnio de
despertar precoz.
c) El abandono del autocuidado hace que estos pa-
cientes tengan un aspecto caracterstico.
d) La aparicin de delirios no es compatible con el
diagnstico de depresin mayor, lo que debe ha-
cernos pensar en trastornos psicticos, como la
esquizofrenia.
e) A diferencia de la depresin neurtica, en la de-
presin mayor el estado de nimo es mejor por la
tarde que por la maana.
45. Seale la respuesta INCORRECTA en relacin
con los trastornos de la personalidad:
a) Se caracterizan por un patrn de comportamiento
rgido y constante, estable a lo largo del tiempo,
que acaban por producir perjuicio al sujeto.
b) El sujeto es capaz de reconocer sus peculiarida-
des, pero no le resultan desagradables.
c) Los pacientes con trastorno obsesivo de la perso-
nalidad suelen ser extrovertidos y con facilidad
para expresar sus emociones.
d) Aunque el tratamiento clsico es la psicoterapia,
algunos pueden responder a psicofrmacos, como
el trastorno lmite.
e) En el trastorno paranoide de la personalidad es ca-
racterstica la desconanza.
46. El litio forma parte de los llamados frmacos esta-
bilizadores del estado de nimo, y se ha utilizado
con frecuencia en el trastorno bipolar. Seale cul
de las siguientes reacciones adversas no asociara
con este frmaco:
a) Hiperparatiroidismo.
b) Hipotiroidismo.
c) Poliuria y polidipsia.
d) Hepatotoxicidad.
e) Temblor de intensidad variable.
47. Paciente varn de 54 aos, con gran frialdad y dis-
tanciamiento emocional, indiferencia a la crtica o
a la aprobacin y con un contacto social muy es-
caso. Forma parte de una secta y practica rituales
de espiritismo, para comunicarse con santos y con
demonios, y sus principales intereses son el vud
y la astrologa. Los pocos que le conocen arman
que siempre ha sido as: solitario y lleno de excen-
tricidades. Seale el diagnstico ms probable:
a) Trastorno delirante crnico (paranoia).
b) Trastorno esquizoafectivo.
c) Trastorno esquizoide de la personalidad.
d) Esquizofrenia paranoide.
e) Trastorno esquizotpico de la personalidad.
48. Sealar el trastorno de la personalidad que se ca-
racteriza por una extrema introversin, tendencia
al aislamiento, frialdad emocional y desinters por
el contacto social:
a) Trastorno antisocial de la personalidad.
b) Trastorno paranoide de la personalidad.
c) Trastorno evitativo de la personalidad.
d) Trastorno esquizoide de la personalidad.
e) Trastorno esquizotpico de la personalidad.
49. Un compaero le habla sobre un paciente sobre el
que duda si padece o no esquizofrenia. Le describe
brevemente sus manifestaciones clnicas. Entre las
booksmedicos.org
-10-
SI MULACRO 1
siguientes, cul sera la ms especca de esqui-
zofrenia?
a) Recientemente, el paciente ha manifestado inters
por la magia negra u otras formas de ocultismo.
b) En ocasiones, el paciente oye voces que repiten
los pensamientos del paciente en voz alta.
c) Sus relaciones sociales han ido a menos progresi-
vamente.
d) Su lenguaje se ha empobrecido en relacin con
entrevistas previas.
e) Tiene cierta dicultad para mantener la atencin.
50. Un paciente de 70 aos, diabtico, es ingresado por
una pielonefritis complicada. Durante su ingreso se
realiza interconsulta a Psiquiatra por un trastorno
uctuante del nivel de conciencia, desorientacin e
incapacidad para jar la atencin. La enfermera de
la planta le explica que, por las noches, dice cosas
raras y est mucho ms inquieto, mientras que por
el da est ms tranquilo y con tendencia al sueo.
Este problema persiste desde hace 48 horas y nunca
antes le haba sucedido nada semejante. Cul sera
el diagnstico ms probable?
a) Amnesia global transitoria.
b) Sndrome orgnico cerebral agudo.
c) Empeoramiento de una demencia tipo Alzheimer
no diagnosticada.
d) Paranoia aguda.
e) Trastorno de ansiedad.
51. Ante un paciente comatoso, con pupilas miticas,
con antecedentes de abuso de drogas ilegales y en
el que se han descartado lesiones en protuberancia
mediante resonancia nuclear magntica, seale el
diagnstico ms probable:
a) Botulismo.
b) Abuso de cocana.
c) Sobredosis de herona.
d) Sndrome de Horner.
e) Intoxicacin por LSD.
52. Juana es una paciente diagnsticada desde hace
1 ao de depresin mayor. Su marido acude con
ella a Urgencias, reriendo que en los ltimos das
ha comenzado a apreciar una conducta diferente a
lo habitual, en la que la paciente reere haber co-
menzado a escuchar voces que insisten una y otra
vez en que lo mejor que puede hacer, dado que no
ya no vale para nada, es quitarse la vida. Dado
el elevado riesgo de suicidio que presenta esta pa-
ciente, adems del ingreso hospitalario, la terapia
electroconvulsiva se nos presenta como una opcin
teraputica interesante por su rapidez de accin.
El efecto secundario ms frecuente de la terapia
electroconvulsiva es:
a) Delirios.
b) Alucinaciones.
c) Trastornos de la memoria.
d) Infarto agudo de miocardio.
e) Cuadros confusionales.
53. Tenemos en la consulta una nia de 18 meses, que
presenta distensin abdominal, episodios de dia-
rrea y, segn reeren los padres, una irritabilidad
general, que se acenta en el momento de las co-
midas. Asimismo, en las tablas de crecimiento est
por debajo del percentil 3. Se le realizan pruebas
serolgicas, en las que presenta niveles elevados de
anticuerpos antirreticulina y antitransglutamina-
sa. Ante la sospecha clnica y analtica de enferme-
dad celaca se decide realizar una biopsia. Seale
el dato anatomopatolgico que NO se encuentra
en el estudio anatomopatolgico intestinal de la
enfermedad celaca:
a) Vellosidades cortas y aplanadas.
b) Criptas profundas.
c) Vacuolas en el epitelio supercial.
d) Inltracin leucocitaria en capas profundas.
e) Material PAS positivo en los macrfagos de la l-
mina propia.
54. Seale cul de las situaciones que se mencionan a
continuacin NO obedece a un trastorno del tbu-
lo distal:
a) Incapacidad para eliminar orina de pH inferior
a 6.
b) Insensibilidad renal a la hormona antidiurtica.
c) Sndrome de Liddle.
d) Sndrome de Fanconi.
e) Seudohipoaldosteronismo.
55. El volumen-minuto cardaco se puede denir
como:
a) Producto de la frecuencia cardaca por el volumen
sistlico.
b) Volumen total de sangre expulsada por el co-
booksmedicos.org
-11-
SI MULACRO 1
razn por metro cuadrado de superficie corpo-
ral.
c) Suma de volumen sistlico ms volumen teledias-
tlico.
d) Trabajo cardaco.
e) Resistencia vascular perifrica multiplicada por
frecuencia cardaca.
56. Mara es una mujer de 29 aos de edad, embara-
zada de 28 semanas, que debuta con un cuadro cl-
nico de dolores musculares generalizados, con es-
calofros y sensacin distrmica, sin foco aparente.
Tras 10 das de evolucin, se aade al cuadro clni-
co un dolor a nivel de la faringe, con cuadro febril
de hasta 38,8C, cefalea y adenopatas cervicales.
Se diagnostica de un cuadro mononuclesico, con
germen identicado como citomegalovirus. Res-
pecto a la infeccin congnita por CMV, son todas
correctas, EXCEPTO:
a) La mayora de los infectados intratero estn asin-
tomticos al nacimiento.
b) La excrecin de CMV en orina durante la 1 se-
mana de vida en presencia de sntomas sospe-
chosos es diagnstico de infeccin congnita.
c) La microcefalia, calcicaciones periventriculares
y la coriorretinitis son los hallazgos ms caracte-
rsticos de la forma sintomtica.
d) La IgG positiva anti-CMV en el suero del RN es
diagnstica del proceso.
e) Es causa frecuente de sordera y retraso mental.
57. Un lactante de 6 meses que, estando previamente
bien, comienza bruscamente con vmitos asocia-
dos a episodios de llanto intenso, palidez y heces
en jarabe de grosella, presenta como diagnsti-
co ms probable:
a) Gastroenteritis vrica.
b) Apendicitis aguda.
c) Clicos del lactante.
d) Invaginacin intestinal.
e) Diarrea secretora.
58. Embarazada de 40 semanas y 5 das de gestacin,
secundpara, con rotura espontnea de bolsas
e inicio del trabajo de parto hace 12 horas, que
tras realizar anestesia raqudea presenta freno del
trabajo de parto, con signos de sufrimiento fetal
(DIPs tipo II y pH de microtoma de sangre fetal de
7,15), siendo necesario practicar una cesrea. En
el quirfano se procede a la valoracin del recin
nacido, que presenta un valor en el test de Apgar
a los 5 minutos de 9 puntos y de 10 puntos a los
10 minutos. En la valoracin de Apgar del recin
nacido se valoran los siguientes parmetros, EX-
CEPTO:
a) Color.
b) Respuesta a la introduccin de una sonda por las
fosas nasales.
c) Frecuencia respiratoria.
d) Tono muscular.
e) Frecuencia cardaca.
59. Un paciente de 65 aos tuvo una pericarditis
aguda hace tres meses. Acude al servicio de ur-
gencias por presentar disnea progresiva desde
hace dos das, junto con signos de bajo gasto
cardaco. El hallazgo de uno de los siguientes
datos exploratorios nos har pensar con ms
seguridad en el diagnstico de taponamiento
cardaco:
a) TA de 100/55 mmHg.
b) Presin venosa central de unos 12 mmHg.
c) Aumento de la presin venosa yugular en unos 5
mmHg con la inspiracin.
d) Disminucin de la PA de 12 mmHg con la inspira-
cin.
e) Soplo sistlico en el borde esternal izquierdo que
no exista previamente.
60. Un recin nacido, hijo de madre diabtica, no ha
expulsado meconio tras 72 horas de vida. Cul es
el cuadro que con ms frecuencia se asocia a esta
situacin?
a) Fibrosis qustica.
b) Aganglionosis rectal.
c) Sndrome de colon izquierdo hipoplsico.
d) Atresia anal.
e) Atresia duodenal.
61. Un RN pretrmino de 29 semanas de edad ges-
tacional y 950 g de peso es alimentado por sonda
nasogstrica y tratado con antibiticos por riesgo
infeccioso. Presenta, a los 12 das de vida, disten-
sin abdominal muy llamativa y heces sanguino-
lentas. Qu prueba le podra dar el signo diag-
nstico de la patologa que debe sospechar?
a) Hemocultivo.
b) Pruebas de coagulacin.
booksmedicos.org
-12-
SI MULACRO 1
c) Hemograma con plaquetas.
d) Radiografa simple de abdomen.
e) Ecografa abdominal.
62. Paciente de 8 aos que acude a su pediatra de
atencin primaria por debilidad crnica. La ex-
ploracin es normal, incluyendo tensin arterial.
En la analtica de rutina destaca un potasio de 2,5
mEq/l y un Mg de 1 mg/dl. En la orina hay hi-
perpotasuria y hipermagnesuria. La aldosterona
y la actividad de renina plasmtica estn elevadas.
Esto es compatible con:
a) Sndrome de Conn.
b) Sndrome de Liddle o pseudohiperaldosteronismo
primario.
c) Sndrome de Bartter.
d) Hiperaldosteronismo por ingesta de regaliz.
e) Parlisis hipopotasmica.
63. Paciente de 61 aos de edad, que es remitido a Ur-
gencias tras haber sufrido una cada al bajar una
escalera. Reere que no ha visto el ltimo escaln,
ha cado con todo el peso sobre el pie izquierdo y
ha perdido el equilibrio, y al caer al suelo se ha
apoyado sobre la mano derecha, golpendose -
nalmente a nivel del hombro derecho en su cada.
De este modo, el paciente presenta tumefaccin
y dolor a la palpacin en hombro y mueca del
lado derecho, as como dolor y tumefaccin en el
tobillo y el pie izquierdo. En el estudio de radio-
loga simple se detecta una luxacin de Lisfranc.
Una luxacin de la articulacin de Lisfranc es una
luxacin:
a) Esterno-clavicular.
b) Acromio-clavicular.
c) Carpo-metacarpiana.
d) Subastragalina.
e) Tarso-metatarsiana.
64. En el proceso de maduracin de los linfocitos T,
los procesos de seleccin positiva y negativa que
implican la restriccin por MHC tienen lugar
en:
a) Mdula sea.
b) Organos linfoides secundarios.
c) Corteza tmica.
d) Mdula tmica.
e) Centro germinal de folculos linfoides.
65. En cul de los siguientes casos es ms probable
que la ictericia de un lactante a trmino sea -
siolgica?
a) Ictericia que aparece a las 12 horas de nacer.
b) Cuando la bilirrubina srica se eleva 5 mg/dl/24
horas o menos en los 2 a 4 primeros das.
c) Cuando la bilirrubina srica directa (conjugada)
es mayor de 1 mg/dl.
d) Ictericia que aparece a los 12 das de nacer.
e) Mxima concentracin de bilirrubina en el 7
da.
66. El genotipo de una determinada clula es:
a) El conjunto de genes de dicha clula, ya estn ac-
tivos o no.
b) El conjunto de los genes que tienen activos una
determinada clula.
c) El conjunto de caractersticas que expresa una c-
lula madura y que la denen como tal.
d) Los diversos tipos de ARNm que pueden transcri-
bir.
e) El conjunto de genes que pueden expresar, sin
considerar los intrones.
67. De los siguientes microorganismos, seale cules
NO provocaran infecciones en la enfermedad
granulomatosa crnica:
a) Aspergillus.
b) Estalococo epidermidis.
c) Neumococo.
d) Serratia marcescens.
e) Candida albicans.
68. Mujer de 57 aos, ama de casa, que presenta desde
hace 2 meses lesiones polimorfas, pruriginosas y
localizadas en el dorso de ambas manos. Adems
presenta edema palpebral bilateral. La paciente
relaciona la aparicin de las lesiones con la com-
pra de un nuevo lavavajillas que se encontraba de
oferta. Sealar la respuesta FALSA con respecto a
su patologa.
a) Si realizramos una biopsia podramos observar
espongiosis al microscopio ptico.
b) Es una dermatitis del ama de casa producida por
sustancias irritantes.
c) El uso de corticoides tpicos y antihistamnicos
orales mejoraran ostensiblemente el cuadro.
booksmedicos.org
-13-
SI MULACRO 1
d) El agente causal ms frecuente en mujeres es el
nquel.
e) Es fundamental retirar el alergeno.
69. Seale cul de los siguientes frmacos NO es til
como antiepilptico:
a) Clonazepam.
b) Primidona.
c) Nortriptilina.
d) Carbamazepina.
e) Etosuximida.
70. Paciente de 16 aos que acude a Urgencias por
disminucin del nivel de conciencia y confusin.
La paciente tiene antecedentes de epilepsia en tra-
tamiento mdico y la familia dice que en el da de
hoy ha tomado una dosis extra de dicho medica-
mento. En la exploracin presenta marcha atxi-
ca, nistagmo, e hirsutismo. Qu medicacin esta-
r tomando?
a) Fenitona.
b) Carbamacepina.
c) Etosuximida.
d) Acido valproico.
e) Trimetadiona.
71. Un paciente de 64 aos ingresa por insuciencia car-
daca izquierda. Se le realiza un ecocardiograma, en
el que destaca la existencia de una fraccin de eyec-
cin normal, un aumento de los espesores de la pared
del ventrculo izquierdo, de predominio en el tabique
del mismo. Uno de los siguientes frmacos NO em-
peorar el estado hemodinmico del paciente:
a) Dobutamina.
b) Nifedipina.
c) Nitroglicerina i.v.
d) Prazosn.
e) Verapamilo.
72. Respecto al delirium, seale la FALSA:
a) Los neurolpticos, los opiceos, el digital o los
salicilatos pueden causarlo.
b) La clnica est muy inuenciada por la personali-
dad previa del paciente.
c) La existencia de un cuadro agitado, con irritabili-
dad, agitacin e inquietud, junto a hiperactividad
simptica, se corresponde con el delirium tre-
mens.
d) La existencia de un cuadro estuporoso, con
letargia, inhibicin e inactividad, sin clnica
vegetativa, se denomina sndrome confusional
agudo.
e) En el caso del delirium tremens es de eleccin el
haloperidol, y respecto al sndrome confusional
agudo, se preeren las benzodiacepinas.
73. Cuando hablamos de pacientes subsidiarios de
recibir cuidados paliativos, cul de los siguien-
tes criterios NO denira a un paciente oncolgico
terminal?
a) Esperanza de vida limitada.
b) Presencia de sntomas que requieren manejo espe-
cco.
c) Complicaciones irreversibles en progresin.
d) Enfermedad oncolgica en fase estable o de se-
cuelas.
e) Los tratamientos con nalidad curativa no estn
indicados.
74. Paciente de 32 aos con disnea leve. Analtica-
mente muestra una anemia microctica e hipo-
crmica con creatinina de 1,8 mg/dl y altera-
ciones en el sedimento urinario. La Rx de trax
muestra un patrn alveolar bilateral y tiene un
patrn funcional en el rango de la normalidad,
salvo una capacidad de difusin de monxido de
carbono (DLCO) de 155%. De las siguientes de-
terminaciones, cul NO es til para el proceso
diagnstico?
a) Determinacin de niveles de c-ANCA.
b) Anticuerpos antimembrana basal glomerular.
c) Anticuerpos antinucleares (ANA).
d) Niveles de inmunoglobulina.
e) Niveles de protena C reactiva.
75. Un paciente de 64 aos, que presenta entre sus
antecedentes personales un adenocarcinoma
de prstata en tratamiento con hormonotera-
pia desde hace seis aos, acude al servicio de
urgencias donde se le realiza un estudio radio-
grfico simple que demuestra la existencia de
metstasis seas que afectan sobre todo a co-
lumna lumbar, presentando clnicamente una
ciatalgia bilateral. Desde un punto de vista fi-
siopatolgico, ese cuadro doloroso lo clasifica-
ramos como:
booksmedicos.org
-14-
SI MULACRO 1
a) Dolor nociceptivo.
b) Dolor seo.
c) Dolor neuroptico.
d) Dolor visceral.
e) Dolor somtico.
76. Gestante de 32 semanas que acude al servicio de
Urgencias con sangrado vaginal oscuro y escaso,
al que precedieron molestias a nivel del hipogas-
trio que han ido aumentando. A la palpacin ab-
dominal se aprecia una marcada hipertona uteri-
na, dolorosa. No se aprecia latido fetal. El estado
materno es malo, con signos de shock y analtica
sangunea de Urgencias compatible con el inicio
de una coagulopata (plaquetopenia, hipobrino-
genemia, aumento del TTPa, disminucin de la
actividad de protrombina). Qu diagnstico le
sugerira?
a) Placenta previa sangrante.
b) Abdomen agudo de causa no obsttrica.
c) Desprendimiento prematuro de placenta.
d) Amenaza de parto pretrmino.
e) Tumoracin sangrante del canal del parto.
77. En relacin con los cambios metablicos que se
producen a lo largo del embarazo, seale cul de
las siguientes armaciones es FALSA:
a) El lactgeno placentario tiene accin antiinsulni-
ca, y favorece la hiperglucemia y la liplisis.
b) El calcio, fsforo, hierro y magnesio se encuen-
tran descendidos.
c) Aumenta el colesterol.
d) En mujeres diabticas, durante la primera mitad
del embarazo, existe un aumento de las necesida-
des de insulina.
e) Aumenta la fosfatasa alcalina.
78. Una joven de 21 aos acude por presentar una
tumoracin no dolorosa de 1 cm, mvil, lisa, sin
jacin a la piel en el cuadrante superoexterno de
la mama izquierda. No se palpan adenopatas. In-
dique el diagnstico ms probable:
a) Necrosis grasa.
b) Fibroadenoma.
c) Carcinoma.
d) Galactocele.
e) Ectasia del conducto mamario.
79. Despus de un expulsivo normal y tras 60 mi-
nutos de perodo de alumbramiento, no se
aprecian signos de desprendimiento placenta-
rio, a pesar de haberse aplicado masaje uteri-
no y de haber incrementado moderadamente
la dosis de oxitocina. Se indica una extraccin
manual de placenta, que resulta imposible por
no existir plano de separacin entre la placenta
y la pared uterina. Cul es el diagnstico ms
probable?
a) Placenta incarcerada.
b) Engatillamiento placentario.
c) Placenta succenturiata con cotiledn aberrante.
d) Placenta adherente por acretismo placentario.
e) Placenta circunvalata.
80. Primpara, 40 semanas de amenorrea, con moni-
torizacin interna intraparto. Se aprecian desace-
leraciones DIP tipo II con todas las contracciones.
Se decide realizar una microtoma de sangre fetal
para analizar el pH. Obtenemos un resultado de
7,14. Su dilatacin es de 6 cm. Cul sera su ac-
titud?
a) Esperar 5-10 minutos y repetir la prueba para con-
rmar un resultado tan extrao.
b) Esperar a que se ponga en dilatacin completa y
hacer el parto utilizando frceps si es preciso, para
as abreviar el expulsivo.
c) Cesrea urgente.
d) Utilizar un tocoltico, para que el feto deje de su-
frir con las contracciones, y posteriormente reali-
zar cesrea.
e) El pH est dentro de la normalidad y, por tanto, no
se tomar ninguna actitud en este momento.
81. Paciente de 42 aos con carcinoma de cuello
diagnosticado por biopsia que invade el labio
anterior y 1 cm de la cpula vaginal; los para-
metrios no estn invadidos; la rectoscopia, la
urografa descendente y la cistoscopia son nor-
males. En este caso, el tratamiento ms correcto
sera:
a) Radioterapia externa ms quimioterapia.
b) Radioterapia de contacto ms quimioterapia.
c) Histerectoma total ampliada tipo Wertheim-Meigs
ms radioterapia externa.
d) Radioterapia de contacto, seguida de histerecto-
ma total simple.
e) Ninguna de las anteriores.
booksmedicos.org
-15-
SI MULACRO 1
82. Paciente de 30 aos, nuligesta, con ltima regla
hace 20 das, que acude a la consulta por presen-
tar esterilidad y dismenorrea importante. En las
exploraciones que se realizan, destaca tumoracin
anexial en la ecografa y elevacin moderada de Ca-
125 srico. Cul es el diagnstico de presuncin?
a) Embarazo ectpico.
b) Cncer de ovario.
c) Mioma uterino.
d) Endometriosis.
e) Quiste ovrico funcional.
83. Una mujer de 47 aos de edad acude a la consulta
reriendo sequedad vaginal de varios meses de
evolucin, con relaciones sexuales dolorosas, a lo
que se suma la reciente aparicin de sofocos, as
como un estado de irritabilidad que, segn ella,
se ve empeorado por las dicultades que presenta
para conciliar el sueo. En relacin a las modi-
caciones endocrinas que caracterizan a este pe-
rodo, seale la respuesta INCORRECTA:
a) Durante la menopausia existen niveles elevados
de FSH.
b) Durante la menopausia existen niveles elevados
de LH.
c) Durante la menopausia, el nivel de estrona es ms
elevado que el de estradiol.
d) Durante la menopausia, los niveles de testos-
terona de origen ovrico disminuyen drstica-
mente.
e) Durante la menopausia, las obesas tienen mayor
produccin de estrgenos perifricos.
84. Doa Mara es una paciente mujer de 53 aos de
edad, con antecedentes personales de mastopata
broqustica e hipertensin arterial, que acude a
la consulta presentando una tumoracin en mama
derecha, con presencia de adenopatas axilares ho-
molaterales. Tras la realizacin de una ecografa
(con signos ecogrcos de malignidad) y el mar-
cado de la lesin mediante arpones, se realiza una
biopsia de la misma que conrma la presencia de
un adenocarcinoma ductal inltrante. Se realiza
exresis de la tumoracin, as como linfadenecto-
ma axilar. El estudio anatomopatolgico del tu-
mor conrma la existencia de receptores estrog-
nicos positivos, por lo que se inicia tratamiento con
tamoxifeno. El tamoxifn tiene un marcado efecto
antiestrognico. Paradjicamente puede producir:
a) Tumores epiteliales de ovario.
b) Endometriosis.
c) Hiperplasia o cncer de endometrio.
d) Miomatosis uterina.
e) Mastopata broqustica.
85. En una mujer embarazada de 18 semanas, sin
antecedentes mdicos de inters, con correcto
seguimiento de la gestacin y sin patologa de
inters durante las primeras 18 semanas, con
niveles de beta-HCG dentro de la normalidad,
PAPP-A dentro de la normalidad, glucoprote-
na b-1 especfica de la gestacin en valores de
normalidad, detectamos una alfafetoprotena
srica elevada. Cul es la malformacin con-
gnita que ms probablemente detectemos en
el feto?
a) Anencefalia.
b) Onfalocele.
c) Mielomeningocele.
d) Gastrosquisis.
e) Atresia de esfago.
86. Para hacer frente de forma ecaz a infecciones por
grmenes intracelulares, que pueden escapar a la
accin de las inmunoglobulinas, as como para re-
solver infecciones de determinados agentes, como
virus u hongos, que difcilmente pueden ser erra-
dicados por la accin de las inmunoglobulinas (de
forma aislada o conjunta a la del sistema de com-
plemento), tenemos la inmunidad celular. Respec-
to a las respuestas de inmunidad celular, indique
la armacin CORRECTA:
a) Las clulas NK siempre expresan, constituti-
vamente, el receptor de afinidad media para
IL-2.
b) Los linfocitos colaboradores TH1 frenan las re-
acciones de inmunidad celular una vez vencida la
infeccin.
c) Los linfocitos colaboradores TH2 controlan y po-
tencian las reacciones de inmunidad celular.
d) En su proceso de maduracin en el timo, los ti-
mocitos que solo interaccionan con el HLA-I se
convierten en LT-CD4.
e) En su proceso de maduracin en el timo, los timo-
citos capaces de interaccionar con el HLA-II se
convierten en LT-CD8.
87. En un paciente en la edad media de la vida con co-
municacin interauricular tipo ostium secundum
con cortocircuito izquierda-derecha mayor de 2:1
y sin signos de hipertensin arterial pulmonar, el
tratamiento de eleccin es:
booksmedicos.org
-16-
SI MULACRO 1
a) Revisiones mdicas peridicas hasta que aparez-
can signos de hipertensin pulmonar.
b) Quirrgico.
c) Ciruga solo cuando existe brilacin auricular.
d) Vasodilatadores.
e) Anticoagulacin oral crnica.
88. Paciente varn de 50 aos que acude a la consulta
del cardilogo por presentar disnea de moderados
esfuerzos desde hace dos meses sin referir sinto-
matologa acompaante. En la exploracin fsica
encontramos FC regular, TA de 130/90 mmHg,
auscultacin: murmullo vesicular conservado
sin estertores y ritmo cardaco regular con soplo
diastlico de pequea intensidad en foco mitral
sin irradiacin. Radiografa de trax: normal.
ECG: estenosis mitral leve, con rea calculada de
1,6 cm
2
, con anatoma valvular conservada. Qu
tipo de tratamiento propondra a este paciente?
a) Comisurotoma mitral abierta, dado que la anato-
ma valvular es favorable.
b) Valvuloplastia percutnea con baln, dado que la
anatoma valvular es favorable.
c) Seguimiento ecocardiogrco peridico con pro-
laxis de endocarditis infecciosa.
d) Sustitucin valvular por prtesis mecnica, dado
que el paciente ya est asintomtico.
e) Tratamiento mdico con dieta hiposdica y diurti-
cos para disminuir la congestin pulmonar, prolaxis
de endocarditis infecciosa y seguimiento peridico.
89. Un joven de 28 aos presenta desde hace dos das
un intenso dolor en la regin retroesternal, que se
incrementa con la inspiracin profunda. No tiene
antecedentes cardiolgicos ni factores de riesgo
coronario conocidos. En cuanto a la exploracin
fsica, la TA es de 135/85 mm Hg y la temperatura
de 37,8 C, siendo normal la auscultacin cardio-
pulmonar. En el electrocardiograma, aparece el
ST elevado con concavidad superior en prctica-
mente todas las derivaciones. Cul es, con ms
probabilidad, la causa del dolor de este paciente?
a) Diseccin de aorta.
b) Pericarditis aguda.
c) Infarto agudo de miocardio extenso.
d) Angina de Prinzmetal.
e) Rotura espontnea del esfago o sndrome de
Boerhaave.
90. Un paciente de 38 aos acude a Urgencias con
dolor centrotorcico de caractersticas anginosas.
Por estudios realizados se concluye que padece
angina de Prinzmetal. Qu tratamiento estara
CONTRAINDICADO?
a) Nitroglicerina sublingual.
b) Nifedipina.
c) Betabloqueantes.
d) Mononitrato de isosorbide.
e) Antiacoagulantes orales.
91. Una mujer de 69 aos, fumadora y con ligera hi-
perglucemia, se queja de molestias torcicas al su-
bir cuestas. Seale cul de las siguientes medidas
NO est indicada inicialmente:
a) Anamnesis completa sobre las caractersticas del
sntoma.
b) Auscultacin cardiopulmonar.
c) Hacer un electrocardiograma (ECG).
d) Solicitar un Holter.
e) Solicitar una prueba de esfuerzo con istopos si el
ECG basal muestra alteraciones de la repolarizacin.
92. En un paciente con dolor torcico sospechoso
de angina de pecho cuya prueba de esfuerzo no
es diagnstica o no es interpretable, por pre-
sentar bloqueo de rama izquierda o hipertrofia
ventricular izquierda, cul de los siguientes es
el mtodo de eleccin para demostrar la isque-
mia?
a) La gammagrafa miocrdica con pirofosfato de
tecnecio.
b) La gammagrafa miocrdica con talio-201.
c) La monitorizacin con Holter durante 24 horas.
d) La estimulacin auricular.
e) La ecocardiografa bidimensional.
93. Paciente de 62 aos de edad, sin antecedentes mdi-
cos de inters, acude a Urgencias por dolor torcico
de 1 hora de evolucin, con irradiacin a mandbu-
la, con severo cortejo vegetativo, que se inici mien-
tras estaba caminando con su mujer. Se solicitan
enzimas cardacas y se realiza un electrocardiogra-
ma, que pone de maniesto la aparicin de onda Q
y elevacin del segmento ST en las derivaciones II,
III y aVF. La troponina presenta valores muy eleva-
dos. Dadas las caractersticas mdicas del paciente
y la corta evolucin temporal del IAM se decide la
reperfusin de las arterias coronarias afectadas. Al
informar a la familia del paciente de las posibles
complicaciones del IAM, estn especialmente inte-
booksmedicos.org
-17-
SI MULACRO 1
resados en conocer las posibilidades de insuciencia
cardaca post-IAM, ya que reeren que el padre del
paciente falleci como consecuencia de una insu-
ciencia cardaca.Cul de estos factores NO au-
menta el riesgo de insuciencia cardaca despus de
un infarto agudo de miocardio?
a) Infarto que abarca el 40% del ventrculo izquierdo.
b) Ancianidad.
c) Diabetes mellitus.
d) IAM previo.
e) Sexo masculino
94. En cul de estas situaciones est CONTRAINDI-
CADO el uso de anticoagulantes?
a) Trombosis venosa profunda.
b) Angina inestable.
c) Prtesis valvular metlica.
d) Pericarditis.
e) Fibrilacin auricular crnica en el seno de una es-
tenosis mitral.
95. La neuralgia del trigmino se caracteriza por los
fenmenos siguientes, EXCEPTO:
a) Se desencadena al tocar la zona gatillo.
b) Es una sensacin constante y unilateral en la re-
gin facial inervada por el trigmino.
c) El tratamiento de eleccin es la carbamacepina.
d) La exploracin fsica es normal.
e) Con mayor frecuencia afecta a las ramas segunda
y tercera.
96. Cul de estas proposiciones es FALSA respecto al
sndrome de West?
a) Se caracteriza por espasmos mioclnicos, retraso
psicomotor e hipsarritmia en el EEG.
b) Se puede tratar con vigabatrina.
c) La enfermedad comienza despus de los 5 aos.
d) Puede ser idioptica.
e) En ocasiones es producida por una encefalopata
metablica, malformaciones cerebrales, secue-
las de meningitis, anoxia perinatal o facomato-
sis.
97. Todas las armaciones respecto al pequeo mal
son ciertas, EXCEPTO:
a) Se inicia en la infancia.
b) Se maniesta por una interrupcin brusca de la
actividad.
c) Puede presentarse tan a menudo como 50 veces al
da.
d) La hiperventilacin disminuye la frecuencia de
estos accesos.
e) En el electroencefalograma se observan complejos
punta-onda a una frecuencia de tres por segundo.
98. Mujer de 20 aos que consulta por prdida de vi-
sin y dolor ocular derecho que aumenta con los
movimientos oculares, de 24 horas de evolucin.
El examen del fondo del ojo es normal y la agude-
za visual est disminuida en ojo derecho. Seale el
enunciado FALSO:
a) La paciente padece una neuritis ptica retrobulbar.
b) Se practic una RM cerebral que fue normal.
c) El pronstico en cuanto a la recuperacin de la
agudeza visual es bueno.
d) El tratamiento indicado es prednisona va oral.
e) La paciente tiene probabilidades de desarrollar
una esclerosis mltiple.
99. Los aneurismas cerebrales pueden ser divididos
de forma general en dos tipos, con diferencias en
cuanto a su localizacin y la frecuencia en que
estos son responsables de la aparicin de una he-
morragia subaracnoidea por su ruptura: los aneu-
rismas saculares y los fusiformes. El 80% de las
hemorragias subaracnoideas que acontecen en la
edad media de la vida se producen por la ruptura
de aneurismas de tipo sacular, que se localizan con
mayor frecuencia en:
a) Arteria comunicante anterior y cerebral anterior.
b) Arteria cerebral media.
c) Arteria cartida interna y comunicante posterior.
d) Arteria cerebral posterior.
e) Arteria cerebelosa posteroinferior.
100. En un paciente de 15 aos con clnica de meningismo,
la existencia de una depresin puntiforme en lnea
media lumbosacra con un angioma e hipertricosis
adyacentes, hace pensar como primer diagnstico en:
a) Diastematomielia.
b) Sinus pilonidal.
c) Teratoma sacro.
d) Seno drmico congnito.
e) Meningitis en un paciente con SIDA.
booksmedicos.org
-18-
SI MULACRO 1
101. Varn de 46 aos, con antecedentes de dolor lum-
bar de repeticin, que ceda al tratamiento con
AINEs y calor seco local, que reere desde hace
varios meses dolor lumbar bajo, con contractura
de la musculatura paravertebral, irradiado por
ambos miembros inferiores por la cara posterior,
desde la regin gltea hasta la planta del pie en la
pierna derecha y hasta la pantorrilla en la pierna
izquierda. El dolor aumenta con la hiperexten-
sin del tronco. Asimismo, reere una sensacin
de hormigueo en ambas piernas siguiendo una
distribucin similar a la del dolor. Adems, nos
cuenta que cuando camina 50 metros tiene que
detenerse por falta de fuerza en las piernas. A la
exploracin, adems de una positividad marcada
para los signos de Lasegue y Bragard a 10 de ele-
vacin, los pulsos en ambos miembros inferiores
estn presentes. Usted lo diagnosticar de:
a) Hernia discal L5-S1.
b) Insuciencia venosa de miembros inferiores.
c) Sndrome de cola de caballo.
d) Arteriosclerosis de miembros inferiores.
e) Estenosis del canal lumbar.
102. Un paciente de 60 aos presenta episodios breves
de dolor lancinante a nivel de hemimandbula de-
recha desencadenados por la masticacin. El exa-
men odontolgico, as como una RM craneal y el
anlisis citolgico del LCR, fueron normales. Qu
tratamiento indicara como primera eleccin?
a) Baclofn.
b) Carbamacepina.
c) Propranolol.
d) Amitriptilina.
e) Termocoagulacin del ganglio de Gasser.
103. Un paciente de 60 aos presenta una crisis genera-
lizada. Cul de las siguientes es la etiologa ms
probable?
a) Tumor.
b) Postraumtica.
c) Idioptica.
d) Enfermedad cerebrovascular.
e) Alcoholismo.
104. En aproximadamente el 50% de las hemorra-
gias intracerebrales lobares existe algn proce-
so patolgico subyacente; el ms frecuente de
ellos es:
a) Tumor primario o metastsico.
b) Anticoagulacin.
c) Angiopata amiloide.
d) Malformacin arteriovenosa.
e) Hipertensin arterial.
105. Un paciente de 63 aos, con diabetes mellitus tipo
2 en tratamiento con insulina, presenta un cuadro
compatible con un temblor esencial, que le llega
a incapacitar para la escritura y la alimentacin
autnoma. Seale el tratamiento de eleccin para
este paciente:
a) Propranolol.
b) Amantadina.
c) Trihexifenidilo.
d) Primidona.
e) Levodopa.
106. Aunque hay una serie de caractersticas que nos
ponen sobre la pista de un parkinsonismo far-
macolgico (simetra, rpida instauracin de los
sntomas, forma acineto-rgida o la aparicin
de acatisia), la gran variedad de las formas de
presentacin hace que ante todo sndrome par-
kinsoniano deba plantearse un posible origen
farmacolgico. Uno de los siguientes frmacos
NO ha sido implicado en el desarrollo de par-
kinsonismo:
a) Haloperidol.
b) Flunaricina.
c) Litio.
d) Pergolide.
e) Reserpina.
107. En un enfermo de 68 aos con artritis reumatoide
que comienza con un cuadro de hipertonia en los
cuatro miembros, exaltacin de los reejos osteo-
tendinosos y aparicin de reejos patolgicos tipo
Babinski, usted pensara en:
a) Cordoma.
b) Polineuritis.
c) Compresin medular por fractura patolgica.
d) Luxacin atlantoaxoidea.
e) Linfoma medular.
108. Seale la respuesta INCORRECTA acerca de los
sesgos de informacin:
booksmedicos.org
-19-
SI MULACRO 1
a) La clasicacin incorrecta diferencial se produce
cuando clasicas mal a uno de los dos grupos de
estudio.
b) La clasicacin incorrecta diferencial se produce
cuando clasicas mal a ambos grupos.
c) El sesgo de atencin se produce cuando el partici-
pante en un estudio modica su comportamiento
al saberse observado.
d) El sesgo de memoria se produce en los estudios
de casos y controles, ya que los casos estn ms
motivados para recordar posibles antecedentes.
e) Como consecuencia de este tipo de sesgos, los
grupos de estudio resultan no comparables.
109. En relacin a las medidas de dispersin, seale lo
FALSO:
a) Son indicadoras del grado de homogeneidad con
que los valores se agrupan alrededor de las medi-
das de tendencia central.
b) El coeciente de variacin es adimensional.
c) La desviacin estndar es la raz cuadrada positi-
va de la varianza.
d) EL rango muestral es la diferencia entre el valor
mximo y mnimo de la serie.
e) Las unidades de medida de la varianza son las
mismas que las de la variable que se describe.
110. Un estudio realizado en una muestra de 90 indi-
viduos con infarto agudo de miocardio evidenci
una colesterolemia media de 240 mg/dl. Cul
ser el mejor ndice de dispersin para estimar la
verdadera colesterolemia media en individuos que
sufren infarto?
a) Desviacin tpica.
b) Desviacin media.
c) Coeciente de variacin.
d) Rango.
e) Error estndar de la media.
111. Un joven de 19 aos de edad, durante una excursin
en el campo, se cort con una lata de conserva aban-
donada en el antebrazo. La herida es profunda y el
lugar donde se encontraba la lata es habitual de paso
de ganado. En el tratamiento de la lesin que presen-
ta este joven es esencial la prolaxis de una enferme-
dad cuya epidemiologa se caracteriza por:
a) El contacto con el suelo es la forma principal de
adquisicin de la enfermedad.
b) Su padecimiento signica la aparicin de inmuni-
dad permanente.
c) El diagnstico de la enfermedad es esencialmente
por laboratorio.
d) La forma clnica ms frecuente es la local.
e) El aislamiento del germen en una lesin indica in-
feccin.
112. Un buen test de screening debe tener todas las si-
guientes caractersticas, EXCEPTO una:
a) Alta sensibilidad.
b) Alta especicidad.
c) Valor predictivo positivo alto, dependiendo de la
prevalencia de la enfermedad.
d) Valor predictivo negativo alto, dependiendo de la
prevalencia de la enfermedad.
e) Seguir la diagonal que va del ngulo inferior iz-
quierdo al superior derecho en las curvas de ca-
ractersticas operativas del receptor.
113. Cul de los siguientes diseos es el ms adecuado
para contrastar una hiptesis etiolgica en una en-
fermedad poco frecuente?
a) Estudio de cohortes.
b) Estudio de casos y controles.
c) Serie de casos.
d) Ensayo clnico aleatorio.
e) Estudio de prevalencia.
114. Como paso siguiente en su plan para la deteccin
precoz de cncer colorrectal, tras investigar la pre-
valencia de la enfermedad realiza Vd. un estudio
para conocer cules son de hecho los valores pre-
dictivos positivo y negativo del test de deteccin de
sangre oculta en heces, en la comunidad sobre la
que trabaja. Los resultados que de este modo ob-
tiene arrojan un valor predictivo positivo del 25%
y un valor predictivo negativo del 98%. Con es-
tos resultados, usted planica a continuacin una
campaa piloto de screening de la enfermedad,
sobre una muestra de la poblacin, para conocer
la aceptacin de este tipo de oferta en su comuni-
dad y estudiar los medios de captacin. Qu ex-
plicara Vd. a las personas que acudiesen, acerca
del signicado de estos valores predictivos positivo
y negativo?
a) Que la probabilidad pre-prueba de tener la enfer-
medad es del 25% y la de no tenerla del 98%.
b) Que si el resultado es positivo, la probabilidad
de no tener la enfermedad es del 2% y, si el re-
booksmedicos.org
-20-
SI MULACRO 1
sultado es negativo, la probabilidad de tener la
enfermedad es del 25%, con lo cual, en uno u
otro caso, sera bastante razonable proseguir es-
tudios.
c) Que de 100 personas con resultado positivo, solo
25 son falsos positivos y, de 100 personas con re-
sultado negativo, 98 son falsos negativos, con lo
cual el rendimiento diagnstico de la prueba es
bastante alto si resulta positiva, mientras que es
demasido bajo si resulta negativa, en cuyo caso se
precisara otro test de screening.
d) Que de 100 personas con resultado positivo solo
25 tienen realmente la enfermedad, debido al in-
conveniente de que la prueba arroja muchos fal-
sos negativos, mientras que si la prueba resulta
negativa, la probabilidad de no tener la enferme-
dad es de 98%, en cuyo caso se puede armar con
bastante seguridad la no existencia de enferme-
dad.
e) Que si la prueba resulta positiva, la probabilidad
de tener efectivamente la enfermedad es solo del
25%, debido al inconveniente de que la prueba
arroja muchos falsos positivos, mientras que si
resulta negativa, se puede armar la no exis-
tencia de enfermedad con una probabilidad del
98%.
115. En un ensayo clnico se comparan tres trata-
mientos (p. ej. placebo, tratamiento establecido
y un tratamiento nuevo). La variable respuesta
es continua (p. ej. nivel de glucosa en sangre).
Aceptando que la variable tiene una distribu-
cin normal, la prueba correcta para comparar
la respuesta es:
a) La t de Student.
b) La de Wilcoxon.
c) El anlisis de la varianza.
d) La de Kruskal-Wallis.
e) La de la Chi-cuadrado.
116. Cul de las siguientes armaciones es FALSA en
relacin con la abilidad y validez de las medidas
clnicas?
a) La abilidad es sinnimo de reproductibilidad y
precisin.
b) La abilidad de las determinaciones de laborato-
rio se establece mediante medidas repetidas.
c) La abilidad de los sntomas puede establecer-
se poniendo de maniesto que son descritos de
manera similar, por diferentes observadores y en
condiciones diferentes.
d) La falta de validez puede relacionarse con un ex-
ceso de error aleatorio.
e) Las condiciones de medicin pueden conducir a
un resultado errneo pero muy able.
117. Al analizar los valores de colesterolemia de un
grupo de 500 pacientes, se observa que esta va-
riable sigue una distribucin normal. Cul de las
siguientes armaciones es FALSA?
a) Los valores de colesterolemia se distribuyen de
forma simtrica.
b) La media de la distribucin y su mediana coinci-
den.
c) La media de la distribucin y su moda no necesa-
riamente coinciden.
d) La media deja la mitad de la distribucin por enci-
ma de su valor.
e) La mediana deja la mitad de la distribucin por
debajo de su valor.
118. En el contraste de hiptesis, una vez establecidas
la hiptesis nula (Ho) y la hiptesis alternativa
(H1), tras jar el valor de p o grado de signica-
cin estadstica, que por convenio suele establecer-
se en 0,05 (el 5%), cul de las siguientes opciones
es FALSA?
a) Si H1 es cierta y no rechazo H0, cometo un error
tipo II.
b) La nica forma de disminuir el error tipo I y II
simultneamente es aumentar el tamao mues-
tral.
c) La potencia del test es la probabilidad de que,
siendo cierta H1, rechace H0.
d) Fijado alfa, si p >alfa, rechazo H0.
e) Si H0 es cierta y la rechazo, cometo el error tipo I.
119. Cul de las siguientes armaciones es FALSA en
relacin con las pruebas diagnsticas?
a) Es preciso seleccionar una prueba sensible cuan-
do las consecuencias de no diagnosticar la enfer-
medad pueden ser importantes.
b) Es razonable comenzar el proceso diagnstico
con pruebas muy sensibles y conrmarlo con tc-
nicas especcas.
c) Una prueba muy especca rara vez es positiva si
no existe enfermedad.
d) Los falsos positivos no son deseables en diagns-
ticos que originen un trauma emocional al sujeto
de estudio.
e) Una prueba sensible es sobre todo til cuando su
resultado es positivo.
booksmedicos.org
-21-
SI MULACRO 1
120. Dos variables presentan un coeciente de correla-
cin (r) de 0,35, que es estadsticamente signica-
tivo (p<0,05). Cul de las siguientes armaciones
es correcta?
a) La correlacin es de tendencia negativa, al ser r
inferior a 0,50.
b) Existe una dbil correlacin.
c) Estudiando ms sujetos, se hubiera obtenido un
coeciente ms elevado.
d) Existen menos de un 5% de probabilidades de que
la asociacin no sea casual.
e) No existe correlacin, ya que el coeciente es me-
nor de 0,50.
121. La realizacin de un programa de actividades pre-
ventivas NO est justicada cuando la enferme-
dad a prevenir tenga:
a) Elevada mortalidad.
b) Prevalencia elevada.
c) Un tratamiento ecaz.
d) Pruebas de cribaje aceptables por la poblacin.
e) Una fase presintomtica corta.
122. Cul de los siguientes es un procedimiento para
el control de factores de confusin en estudios epi-
demiolgicos?
a) Prueba de la Chi-cuadrado.
b) Apareamiento (matching).
c) Anlisis de la varianza.
d) Anlisis por intencin de tratar.
e) Anlisis de subgrupos.
123. Un estudio observacional estudia la probabilidad de
haber alcanzado una respuesta clnicamente signi-
cativa despus de dos aos de tratamiento con un
frmaco A o con un frmaco B en la artritis reuma-
toide. Se realiz un anlisis de regresin logstica uti-
lizando como variable dependiente la existencia de
respuesta clnicamente signicativa a los dos aos
y como variables independientes el grupo de trata-
miento (frmaco A o frmaco B), la edad, el sexo y
el tiempo de evolucin de la enfermedad. Los resul-
tados arrojan una odds ratio de 3,5 y un intervalo de
conanza del 95% de 1,7-4,2 a favor del tratamiento
A(P = 0,0001). Cul de las respuestas es FALSA?
a) Las diferencias a favor del tratamiento A depen-
den de las otras variables incluidas en el anlisis.
b) La posibilidad de obtener una respuesta clnica
signicativa es 3,5 veces superior para el frmaco
A que para el frmaco B.
c) Aunque edad, sexo y tiempo de evolucin de la
enfermedad tambin podran inuir en la conse-
cucin de una respuesta signicativa, los resul-
tados referidos al frmaco son independientes de
estas tres variables.
d) Al tratarse de un estudio no aleatorizado, no se
puede descartar el efecto de otras variables no
consideradas en el anlisis.
e) Aunque el efecto del tipo de tratamiento inuya
signicativamente sobre la probabilidad de alcan-
zar una respuesta clnica, cualquiera de las otras
tres variables podra alcanzar tambin la signi-
cacin estadstica.
124. El mejor parmetro para valorar la reproducibili-
dad de una prueba diagnstica es:
a) Sensibilidad.
b) Especicidad.
c) Valor predictivo negativo.
d) Valor predictivo positivo.
e) Ausencia de dispersin en una serie de medicio-
nes.
125. El benecio conseguido con una prctica asisten-
cial, cuando se aplica en condiciones normales, se
denomina:
a) Calidad.
b) Productividad.
c) Ecacia.
d) Efectividad.
e) Eciencia.
126. La potencia de una prueba estadstica de contraste
de hiptesis corresponde a:
a) La posibilidad de realizar comparaciones mltiples.
b) El grado de independencia de las asunciones so-
bre la distribucin de las variables.
c) El valor complementario de la p.
d) La capacidad para detectar hiptesis alternativas.
e) El nmero de grados de libertad de la prueba.
127. Varn de 30 aos, con una colitis ulcerosa cono-
cida, que viene con signos de colitis grave, ebre
elevada, taquicardia, deplecin de volumen con
booksmedicos.org
-22-
SI MULACRO 1
desequilibrio electroltico y dolor abdominal. La
placa simple de abdomen muestra una dilatacin
del colon de 7 cm de dimetro. Sealar la FALSA:
a) El empleo de loperamida puede haberlo desenca-
denado.
b) La localizacin ms frecuente de la dilatacin es
el colon descendente-sigma.
c) Si se perfora, la mortalidad es superior al 30%.
d) Si al cabo de 48 horas de tratamiento no hay clara
mejora, debe hacerse colectoma de urgencia.
e) Hay que administrar corticoides intravenosos y
antibiticos de amplio espectro.
128. Indique cul de las siguientes armaciones NO le
parece correcta:
a) La hepatitis por el virus D se evita con una vacuna
recombinante que induce la formacin de anti-HBs.
b) El mecanismo de transmisin del virus de la hepa-
titis C no se reconoce hasta en el 40% de los casos
de hepatitis aguda por este virus.
c) La hepatitis fulminante de origen vrico suele ser
causada por el virus tipo C.
d) La coinfeccin aguda por virus B y D no induce
mayor tendencia a la cronicidad que la infeccin
aislada por virus B.
e) La infeccin por el virus C tiende al desarrollo de
la hepatitis crnica y de la cirrosis.
129. Con respecto a las enfermedades vasculares intes-
tinales, seale la opcin INCORRECTA:
a) La colitis isqumica no complicada solo requiere
tratamiento mdico.
b) La aparicin de dolor abdominal clico despro-
porcionado a los signos fsicos en un paciente
cardipata debe hacernos sospechar isquemia me-
sentrica aguda (IMA).
c) Los aneurismas de arteria esplcnica, aunque son
los ms frecuentes, tienen escasa tendencia a la
rotura espontnea.
d) La causa ms frecuente de IMA es la embolia.
e) El tratamiento de la IMA requiere embolectoma o
brinolticos.
130. Varn de 60 aos con EPOC que ha ingresado por
neumona y reagudizacin grave de la insuciencia
respiratoria, precisando tratamiento con esteroides
e ingreso en UCI. En el 5 da de ingreso, el paciente
presenta episodio de hemorragia digestiva alta. Lo
ms probable es que la endoscopia muestre:
a) Erosiones gstricas que afectan a cuerpo y fun-
dus.
b) Erosiones gstricas en antro.
c) Ulcera duodenal.
d) Tumor gstrico.
e) Gastritis atrca.
131. Mujer de 67 aos, fumadora de 20 cigarrillos/
da. Desde hace un mes tiene febrcula vesper-
tina, astenia, anorexia, prdida de 3 kg de peso
y dolor epigstrico continuo que no se irradia
ni se relaciona con las comidas. En los ltimos
10 das ha presentado orinas colricas, con-
juntivas ictricas, heces hipoclicas y prurito
generalizado. Exploracin fsica: tensin arte-
rial 130/75 mm Hg, pulso 75 lpm, temperatu-
ra 36,5 C, ictericia cutaneomucosa, abdomen
blando, depresible, se palpa fondo de la vescu-
la biliar. Analtica: Hb 11 gr, leucocitos 8.000,
protrombina 70%, bilirrubina (total 10,3
mg/100 ml, directa 8,2 mg/100 ml), AST 158
u/l, ALT 105 U/l, GGT 350 U/l, fosfatasa alca-
lina 1154 U/l. Rx abdomen normal. Ecografa
abdominal: vescula distendida sin clculos en
su interior, va biliar intra y extraheptica di-
latada. Ante todo lo anterior, nuestra primera
sospecha sera:
a) Pancreatitis crnica.
b) Cncer de estmago.
c) Cncer de vescula biliar.
d) Ampuloma.
e) Tumor de Klatskin (colangiocarcinoma hiliar).
132. Cul de las siguientes entidades NO es considera-
da como un precursor del cncer gstrico?
a) Anemia perniciosa.
b) Ulcera prepilrica.
c) Plipo adenomatoso gstrico.
d) Gastritis crnica atrca.
e) Enfermedad de Mentrier.
133. En un paciente con espasmo esofgico difuso
puede estar indicado todo lo siguiente EXCEP-
TO:
a) Nitratos.
b) Nifedipino.
c) Cisapride.
d) Trazodona.
e) Dilatacin con baln.
booksmedicos.org
-23-
SI MULACRO 1
134. Una mujer de 45 aos, con cirrosis grado A de
Child por una hepatitis autoinmune, presen-
t un episodio de hemorragia digestiva. En la
endoscopia nicamente se observaron varices
esofgicas con estigmas de hemostasia reciente
sin sangrado activo, por lo que se realiz liga-
dura con bandas. Tras la realizacin de esta te-
raputica, cul de las siguientes afirmaciones
es cierta?
a) Se debe realizar adems escleroterapia.
b) No se recomienda repetir la ligadura con ban-
das.
c) La tasa de mortalidad es superior a la que se da
tras la aplicacin de escleroterapia.
d) El porcentaje de recidiva hemorrgica con este
tratamiento es igual o inferior al que se produce
tras escleroterapia.
e) La perfusin continua de somatostatina tras la co-
locacin de las bandas no previene el desarrollo
de la recidiva hemorrgica.
135. Un varn de 54 aos acude a su consulta por
presentar, desde hace tres meses, dolor en epi-
gastrio irradiado a ambos hipocondrios y prdi-
da de 5 kg de peso en dicho perodo. Las prue-
bas de laboratorio evidenciaron FA 789, GGT
564, resto normal. Se realiz una ecografa que
no mostr alteraciones, presentando la va biliar
un calibre normal. Cul cree que es la prueba
que le aportara ms informacin sobre la etio-
loga?
a) Biopsia heptica.
b) Colangiopancreatografa retrgrada endoscpica.
c) Colangiografa transparietoheptica.
d) TC.
e) RMN.
136. En relacin con los plipos del colon, indique la
armacin que le parece cierta:
a) Hasta el 25% de los plipos malignizan.
b) La mayora de los cnceres colorrectales derivan
de un plipo adenomatoso.
c) El sndrome de Gardner asocia los plipos del
colon a neoplasias del sistema nervioso cen-
tral.
d) La polipectoma con colonoscopia no es trata-
miento suciente de un plipo con carcinoma
in situ (no sobrepasa la membrana basal glan-
dular).
e) Los plipos adenomatosos tubulares son los que
ms tienden a malignizarse.
137. Una mujer de 35 aos, con ingesta de alcohol
importante desde hace aos, acude a Urgencias
por ictericia y malestar general. En la analtica
efectuada se observa lo siguiente: GPT 3.358
mU/ml; GOT 2.855 mU/ml, bilirrubina total 8
mg/dl. Cul de estas afirmaciones es correc-
ta?
a) El pronstico de la hepatopata es grave.
b) La realizacin de una ecografa abdominal carece
de utilidad.
c) Probablemente sufre una hepatitis alcohlica.
d) Probablemente se trata de una ictericia obstructiva.
e) La analtica es sugestiva de necrosis heptica ex-
tensa.
138. Seale la respuesta FALSA en relacin a la infec-
cin por VHC:
a) La mayor parte de los casos de hepatitis aguda por
VHC son asintomticos.
b) El 80% de los casos de hepatitis C evolucionan a
la cronicacin.
c) La transfusin de sangre contaminada repre-
senta el principal mecanismo de transmisin.
d) Los pacientes con infeccin crnica pueden pre-
sentar Ac anti-LKM1 en suero.
e) La transmisin sexual del VHC es rara.
139. Una mujer de 67 aos, con el diagnstico previo
de cirrosis heptica por infeccin por el virus de
la hepatitis C, acude al Hospital por descompen-
sacin hidrpica en forma de ascitis, como haba
tenido en otras ocasiones, pero con dolor abdomi-
nal difuso, que se conrma a la palpacin, y ebre.
Se decide realizar paracentesis diagnstica, cuyo
resultado es el siguiente: LDH 875, glucosa 25,
albmina 0,26, clulas 15.700 con 90% de PMN.
Ante estos resultados, qu respuesta considera
correcta?
a) Se trata de una peritonitis bacteriana espont-
nea, causa frecuente de descompensacin hi-
drpica y se debe iniciar tratamiento con una
cefalosporina de tercera generacin (cefoxitina
o cefuroxina) por va intravenosa.
b) Se trata de una peritonitis bacteriana espont-
nea, causa frecuente de descompensacin hi-
drpica, siendo la causa ms probable el ente-
rococo.
c) Se trata probablemente de una peritonitis bacte-
riana espontnea, del paciente cirrtico, probable-
mente producida por E.coli, y que debe tratarse
con cefalosporinas de tercera generacin.
booksmedicos.org
-24-
SI MULACRO 1
d) Es poco probable que se trate simplemente de una
peritonitis bacteriana espontnea, siendo obliga-
torio que se descarten causas de peritonitis secun-
daria.
e) Habitualmente el lquido asctico de los cirrti-
cos tiene las caractersticas descritas, por lo que
no hay que pensar en infecciones sobreaadi-
das.
140. Paciente de 65 aos, con antecedente de colon irri-
table, que presenta desde hace 6 meses hemato-
quecia y dolor con la defecacin. A la exploracin,
el tacto rectal es normal. Cul sera la actitud
ms adecuada ante este caso?
a) Hormonas tiroideas.
b) CEA.
c) Dieta rica en bras.
d) Enema opaco.
e) Sigmoidocolonoscopia.
141. Un varn de 50 aos acude a consulta reriendo
dolor abdominal epigstrico ocasional y diarrea
de dos aos de evolucin. Se le practica un test de
cuanticacin de grasa en heces de 24 h, determi-
nndose la presencia de esteatorrea. La siguiente
prueba que le realizamos es la prueba de la D-xi-
losa, que es normal. Cul es el diagnstico ms
probable en este paciente?
a) Enfermedad de Crohn.
b) Amiloidosis.
c) Gastritis eosinla.
d) Pancreatitis crnica.
e) Linfoma intestinal.
142. Varn de 28 aos con antecedente familiar de un
hermano afecto de poliposis adenomatosa del co-
lon. Se encuentra en seguimiento con colonosco-
pias bianuales por plipos adenomatosos. En una
analtica de control se demuestra FA 837 UI/l,
FGT 715 UI/l, Bb 1,9 mg/dl, GOT 62 UI/l, GPT
100 UI/l. Los niveles de CEA son de 107 UI/ml.
Con estos datos, el diagnstico ms probable sera
el de carcinoma:
a) Periampular.
b) De colon con metstasis hepticas.
c) Pancretico.
d) Germinal extragonadal.
e) De vescula biliar con metstasis seas.
143. Cul de los siguientes trastornos metablicos es
el responsable de la litiasis renal en un paciente
con sarcoidosis?
a) Hiperuricosuria.
b) Hipercalcemia absortiva.
c) Hipercalciuria resortiva.
d) Oxaluria.
e) Hipocitruria.
144. Un varn de 50 aos consulta por presentar
una lesin exantemtica en el trax que se ex-
pande hasta formar una gran lesin anular de
color rojo vivo que se atena en el centro. Esta
lesin ni le dola ni le molestaba, pero unos das
despus le aparecieron otras lesiones anulares
secundarias, de aspecto similar al de la lesin
inicial, acompaadas de cefalea, ligera rigidez
de nuca, fiebre, escalofros, artralgias y males-
tar y cansancio profundos. Cul es su diagns-
tico?
a) Enfermedad de Lyme.
b) Leptospirosis.
c) Fiebre recurrente.
d) Tularemia.
e) Slis secundaria.
145. En relacin a la hiperplasia benigna de prstata
(HBP), todos los postulados son ciertos, EXCEP-
TO:
a) El estmulo andrognico a travs de su forma
activa, la dihidrotestosterona, ejercera un papel
fundamental.
b) Existe correlacin clara entre la HPB y el cncer.
c) Es la causa ms frecuente de obstruccin del trac-
to urinario inferior en el varn.
d) Puede existir residuo postmiccional secundario a
la obstruccin infravesical.
e) No existe correlacin entre el tamao prosttico y
el grado de obstruccin.
146. Cul es el tratamiento de eleccin, en principio,
de un adenocarcinoma prosttico estadio T2a en
un varn de 60 aos, sin otros problemas de sa-
lud?
a) Prostatectoma radical.
b) Supresin hormonal quirrgica (castracin).
c) Supresin hormonal farmacolgica.
booksmedicos.org
-25-
SI MULACRO 1
d) Quimioterapia.
e) No existe alternativa ecaz de tratamiento (trata-
miento sintomtico).
147. Paciente de 35 aos que consulta por proteinuria y
microhematuria descubiertas en un reconocimien-
to rutinario. Como nico antecedente reere haber
presentado en algunas ocasiones orinas oscuras co-
lor coca-cola, coincidiendo con infecciones amigda-
lares. La funcin renal, TA y complemento srico
son normales, mientras que la tasa de IgA srica
est elevada. Cul es el diagnstico ms probable?
a) Tuberculosis urinaria.
b) GN mesangial IgA.
c) Sndrome de Alport.
d) Prpura de Schonlein-Henoch.
e) GN membranoproliferativa tipo III.
148. Seale cul de las siguientes armaciones es co-
rrecta en cuanto a la enfermedad qustica medular:
a) Cuando se asocia a degeneracin de la reti-
na, la herencia es autosmica recesiva y se
asocia con insuficiencia renal antes de los 20
aos.
b) Hay una dilatacin hasta proporciones qus-
ticas de los conductos colectores, selectiva-
mente en los extremos que llegan a las papi-
las.
c) El anlisis de orina es generalmente anormal,
siendo lo ms caracterstico la hematuria mi-
croscpica y los cilindros granulosos.
d) Es raro que se desarrolle insuficiencia renal, y
cuando aparece, la mejor opcin teraputica es
el trasplante.
e) La hipertensin arterial es un acontecimiento
terminal, que se debe a la dificultad para la eli-
minacin hidrosalina.
149. Un varn est diagnosticado de carcinoma de clu-
las renales. Se le practica una nefrectoma radical,
y en el examen microscpico de la pieza se observa
invasin de la cpsula renal, la grasa perirrenal,
pero connada al interior de la fascia de Gerota.
A qu estadio corresponde?
a) Estadio T1b.
b) Estadio T3b.
c) Estadio T2.
d) Estadio T3a.
e) Estadio T4.
150. Referente al rin esponjoso medular, seale lo
correcto:
a) La mayora de los casos son de herencia autos-
mica recesiva.
b) La incidencia de hipertensin es mayor que en la
poblacin general.
c) La evolucin a IRC es frecuente.
d) Se diagnostican, en general, en los primeros 10
aos de vida.
e) Hay mayor incidencia de clculos e infeccin.
151. El sndrome de Bartter se caracteriza por los si-
guientes aspectos, EXCEPTO:
a) Alcalosis metablica.
b) Hipopotasemia.
c) HTA.
d) Hipomagnesemia.
e) Hiperreninemia.
152. A propsito del antgeno prosttico especco
(PSA), todo es cierto EXCEPTO:
a) No es de utilidad en el diagnstico de reincidencia
despus de la prostatectoma radical.
b) En adultos jvenes, el valor normal se encuentra
entre 0-4 ng/ml.
c) La densidad del PSA (PSA srico/ volumen pros-
ttico) es til en pacientes con prstata grande, en
los que el PSA elevado puede deberse a hiperpla-
sia benigna.
d) Las glndulas prostticas neoplsicas bien dife-
renciadas producen ms PSA que las cancerosas
no diferenciadas.
e) El PSA puede presentar aumentos normales en re-
lacin con la edad.
153. Referente a la ATR tipo II, todo es cierto, EX-
CEPTO:
a) La reabsorcin de bicarbonato es defectuosa.
b) Segn desciende el bicarbonato, la disminu-
cin de la carga filtrada puede alcanzar un ni-
vel en el que el tbulo puede reabsorber, y la
orina queda libre de bicarbonato y tiene un pH
bajo.
c) Se acompaa de hipercalciuria y litiasis.
d) Se trata de una acidosis con anin GAP normal.
e) En la sobrecarga con amonio, el pH urinario baja
de 5,5.
booksmedicos.org
-26-
SI MULACRO 1
154. 19c) Un varn de 10 aos acude por edemas
maleolares. La exploracin fsica es normal y no
existe hipertensin arterial. Analticamente desta-
ca: creatinina (sangre) 0,7 mg/dl, albmina (san-
gre) 2 g/l y proteinuria de 6 g en orina recogida du-
rante 24 horas. El sedimento urinario es normal.
Cmo clasicara sindrmicamente este caso?
a) Sndrome nefrtico.
b) Proteinuria en rango no nefrtico.
c) Sndrome nefrtico.
d) Alteraciones urinarias mnimas.
e) Glomerulonefritis aguda.
155. Un varn de 68 aos con diabetes mellitus tipo II,
de aproximadamente tres aos de evolucin, en
buen control metablico con dieta y al parecer no
hipertenso, en un control realizado dos aos an-
tes, presenta TA de 220/120 mmHg, encontrndo-
se como nicos hallazgos en la exploracin fsica
galope por cuarto tono y frecuentes cruces arte-
riovenosos en el fondo de ojo. La radiografa de
abdomen muestra asimetra de tamao emtre las
siluetas renales, siendo la derecha de aproximada-
mente 8 cm de largo. Qu diagnstico debe sospe-
char en primer lugar?
a) Trombosis de la vena renal derecha.
b) Nefropata diabtica.
c) Pielonefritis crnica unilateral.
d) Estenosis arterioesclertica de la arteria renal de-
recha.
e) Uropata obstructiva.
156. Un enfermo de 60 aos con rin nico debu-
ta con proteinuria progresiva que termina en
sndrome nefrtico orido e insuciencia renal.
Qu lesin esperara encontrar en la biopsia
renal?
a) Glomerulonefritis membranoproliferativa.
b) Glomerulonefritis membranosa.
c) Hialinosis segmentaria y focal.
d) Nefropata por reujo.
e) Enfermedad de cambios mnimos.
157. Una mujer de 40 aos presenta erupcin cutnea
en forma de petequias y ppulas, sobre todo en
los dedos, ebre, artralgias y tenosinovitis. In-
terrogada por sus antecedentes personales, nos
cuenta que es prostituta, fumadora de 20 cigarri-
llos al da, ADVP y bebedora ocasional. Adems
cuenta que hace pocas semanas tuvo un episodio
de escozor intenso al orinar con ujo blanquecino
que achac a algn problema alrgico a los pre-
servativos. Ante esta situacin, seale la respuesta
verdadera:
a) Es muy probable que estemos ante una slis se-
cundaria.
b) El tratamiento de eleccin es la ceftriaxona.
c) El agente causal ms probable es la Chlamydia
trachomatis.
d) Es un sndrome de Fitz-Hugh-Curtis.
e) Es muy til la serologa para el diagnstico.
158. Un enfermo de prtesis valvular cardaca va a ser
sometido a una exploracin mediante colonosco-
pia. Cul de las siguientes le parece la pauta de
prolaxis ms adecuada?
a) Amoxicilina va oral.
b) Ampicilina intravenosa.
c) Eritromicina va oral.
d) Ampicilina y gentamicina intravenosas.
e) Clindamicina intravenosa.
159. Acude a la consulta de Dermatologa un peletero
con una lesin no dolorosa en el antebrazo dere-
cho, indurada, de centro necrtico, rodeado de un
halo eritematoedematoso y una adenopata regio-
nal. Cul cree que es el diagnstico ms proba-
ble?
a) Tularemia.
b) Fiebre botonosa mediterrnea.
c) Fiebre mediterrnea familiar.
d) Enfermedad por araazo de gato.
e) Carbunco.
160. Qu actitud tomara ante un varn de 65 aos,
asintomtico y con buen estado general, que pre-
senta en la placa de trax residuos brosos suges-
tivos de tuberculosis antigua (sin presentar signos
de tuberculosis activa) y que nunca ha recibido
tratamiento antituberculoso?
a) Abstencin teraputica.
b) Observacin y revisin a los 6-12 meses.
c) Quimioprolaxis con isoniacida 300 mg/da 12
meses.
d) Tratamiento con isoniacida 300 mg/da y rifampi-
cina 600 mg/da durante 9-12 meses.
booksmedicos.org
-27-
SI MULACRO 1
e) Estudio de resistencias del M. tuberculosis, eli-
giendo el tratamiento en funcin de ste.
161. Un varn de 65 aos, en tratamiento con azatioprina
por una enfermedad de Crohn, acude por presentar
en la ltima semana ebre diaria, astenia, tos y ex-
pectoracin hemoptoica. En la Rx de trax se evi-
dencia un inltrado micronodular bilateral. Cul
de los siguientes microorganismos le parece MENOS
probable como responsable del cuadro del paciente?
a) M. tuberculosis.
b) P. carinii.
c) P. aeruginosa.
d) C. albicans.
e) CMV.
162. Cul es el animal que transmite habitualmente la
hidatidosis al hombre?
a) Cerdo.
b) Gato.
c) Perro.
d) Ganado vacuno.
e) Cabra.
163. Paciente de 17 aos que acude con cefalea, que au-
menta con la exin del cuello. Tambin le comenta
la disminucin de visin del ojo derecho y que lti-
mamente nota que necesita aumentar el volumen de
la TV o la radio. Cuando le explora la rigidez de nuca
(que est ligeramente aumentada), le observa una pla-
ca de pigmentacin blanca y varios focos craneales de
alopecia. La acumetra demuestra un Rinne positivo
con Weber lateralizado al derecho y un Schwabach
acortado. Realiza una puncin lumbar que muestra
pleocitosis linfocitaria con 25 mg protenas/dl y 58 mg
glucosa/dl. Qu tratamiento realizara?
a) Anfotericina B.
b) Aciclovir.
c) Corticoides y/o azatioprina.
d) Ceftriaxona.
e) Ampicilina+tobramicina.
164. En la leishmaniasis visceral, qu NO es cierto?
a) El diagnstico suele darlo el estudio de mdula sea.
b) La eritromicina es el tratamiento de eleccin.
c) El hallazgo ms llamativo a la exploracin es la
esplenomegalia.
d) La anemia, leucopenia y trombopenia son comunes.
e) Es tpico que exista hipergammaglobulinemia po-
liclonal.
165. El principal mecanismo defensivo frente a Mycobac-
terium tuberculosis es la inmunidad celular, que pue-
de ser puesta de maniesto mediante la prueba de
la tuberculina (reactividad cutnea a la tuberculina).
As, con respecto a la prueba de la tuberculina, cul
de las siguientes armaciones es cierta?
a) Se administra por va i.m.
b) Una prueba de la tuberculina no puede sensibili-
zar a una persona no infectada.
c) La reaccin se considera positiva cuando el rea
de eritema es >10 mm.
d) Puede dar una falsa reaccin positiva en pacientes
con sarcoidosis.
e) La reaccin debe leerse a las 24 horas.
166. Respecto a las siguientes armaciones sobre la in-
feccin en el paciente neutropnico, seale la IN-
CORRECTA:
a) El tratamiento emprico inicial con una nica ce-
falosporina antipseudomonas puede ser adecuado.
b) El ectima gangrenoso est causado de forma ex-
clusiva por Pseudomonas.
c) El sndrome de Sweet puede ser un cuadro para-
neoplsico.
d) En un paciente que contina febril despus de 7
das de tratamiento se debe aadir anfotericina B.
e) La tiitis cursa clnicamente como una apendicitis.
167. Qu armacin es FALSA respecto a los nios
con primoinfeccin tuberculosa?
a) La mayora estn levemente enfermos.
b) La nica evidencia de enfermedad puede ser la
seroconversin del Mantoux.
c) Rara vez se detecta radiolgicamente el complejo
primario.
d) Los nios son muy contagiosos y precisan aislamiento.
e) Precisan tratamiento tras el diagnstico durante
9-12 meses.
168. Referente al VIH, todas las armaciones son co-
rrectas, EXCEPTO una:
booksmedicos.org
-28-
SI MULACRO 1
a) El receptor vrico del VIH es la protena gp 120.
b) Puede infectar otras clulas adems del linfocito T4.
c) Su receptor celular es la molcula CD4.
d) Pertenece al gnero lentivirus.
e) Es un virus DNA.
169. De las infecciones causadas por Listeria monocyto-
genes, NO es cierto:
a) Se transmite por alimentos contaminados.
b) En la meningitis, el LCR suele presentar una hi-
poglucorraquia.
c) Son facilitadas por el tratamiento esteroideo crnico.
d) La mayora de las infecciones del embarazo se de-
tectan en el primer trimestre.
e) La edad avanzada es un factor favorecedor.
170. Paciente de 43 aos, en tratamiento por leucemia
aguda que comenz con ebre sin foco aparente,
y al que se le puso tratamiento con ceftazidima
IV, amikacina IV y vancomicina IV, desarrolla al
cuarto da de tratamiento un rash en tronco, con
elementos eritematosos y pustulares, no dolorosos,
y contina febril. De las siguientes opciones, cul
le parece ms correcta?
a) Cambiar a meropenem i.v.
b) Asociar aciclovir i.v.
c) Asociar anfotericina B i.v.
d) Asociar esteroides i.v. y antihistamnicos v.o.
e) Asociar eritromicina i.v.
171. La aparicin de barro biliar, con clnica ocasional
de colecistitis aguda, es un efecto secundario de:
a) Aztreonam.
b) Ceftriaxona.
c) Gentamicina.
d) Penicilina G.
e) Imipenem.
172. Una mujer de 35 aos, previamente sana, ingres
el pasado agosto por ebre y cefalea intensa. En
la exploracin se encontr un exantema maculo-
papuloso en tronco y extremidades con afecta-
cin de palmas y plantas. En la cara interna del
muslo izquierdo se descubri una lesin ulcerada
necrtica de fondo negruzco. Existan adenopa-
tas inguinales izquierdas y se palpaba el polo del
bazo. Cul es el diagnstico ms probable?
a) Linfogranuloma venreo.
b) Slis secundaria.
c) Eritema multiforme.
d) Lupus eritematoso.
e) Fiebre botonosa mediterrnea.
173. En el complejo mayor de histocompatibilidad, en
humanos, existen molculas de distintas clases.
Cul es la que est ms ampliamente distribui-
da, ya que existe en la prctica totalidad de clulas
nucleadas?
a) Clase I.
b) Clase II.
c) Clase III.
d) Clase IV.
e) Clase V.
174. Qu antibitico acta sinrgicamente junto a eri-
tromicina frente a Legionella?
a) Quinolonas.
b) Penicilina.
c) Rifampicina.
d) Doxiciclina.
e) Trimetoprim-sulfametoxazol.
175. Varn de 24 aos que llega a su consulta tras va-
rios das de ebre y malestar general. El da ante-
rior le aparecieron unas lesiones vesicopustulosas
en la piel y comienza con molestias en la mueca
derecha. En la anamnesis, el paciente recuerda
artralgias migratorias y comenta algn contacto
sexual no habitual en los ltimos meses. Ud. sos-
pechara:
a) Fiebre reumtica.
b) Gonococia generalizada.
c) Slis secundaria.
d) Sndrome de Reiter.
e) Meningococcemia crnica.
176. Mujer de 80 aos que presenta desde hace 5
aos una lesin elevada, erosivocostrosa, de bor-
des mal denidos y sin que se observen perlas
epiteliales. La histologa revela la existencia de
nidos de clulas epidrmicas atpicas que inva-
den la dermis. Ante este cuadro clnico, seale
la FALSA:
booksmedicos.org
-29-
SI MULACRO 1
a) Cuando aparece sobre una lcera crnica, que-
madura o cicatriz, se denominan lcera de Mar-
jolin.
b) Dada la posibilidad de metastatizar, el tratamiento
quirrgico es el de eleccin.
c) Este tipo de cncer aparece no solo en la piel, sino
tambin en las mucosas.
d) En la mano, es el tumor maligno cutneo ms fre-
cuente.
e) Este tipo de tumor no se relaciona con la fotoex-
posicin crnica y s con la aguda.
177. El nivel III de la clasicacin de Clark en el mela-
noma maligno corresponde a una de las siguientes
proposiciones. Indquela:
a) Invasin de la epidermis.
b) Invasin de la dermis papilar parcialmente.
c) Invasin de la dermis papilar totalmente.
d) Invasin de la dermis reticular parcialmente.
e) Invasin de la hipodermis.
178. Varn de 28 aos con lesiones papulopustulosas,
algunas recubiertas con costras, comedones ce-
rrados, elementos qusticos y cicatrices hipertr-
cas queloideas en espalda, hombros y regin
preesternal. Ha realizado diversos tratamientos,
generales y locales, sin resultado. Qu indicara
usted?
a) Acido 13-cis-retinoico.
b) Antipaldicos de sntesis.
c) Hidrazidas.
d) Antihistaminicos.
e) Piperazina.
179. Seale la FALSA con respecto a la enfermedad de
una mujer de 27 aos de edad que presenta n-
dulos dolorosos eritematosos en cara anterior de
piernas, en varios brotes imbricados desde hace 3
meses; no se acompaa de ebre ni de otra sinto-
matologa sistmica:
a) No es precisa la biopsia cutnea, dada la claridad
clnica del cuadro.
b) Posiblemente tome anticonceptivos orales que
pueden ser los causantes.
c) Se debe intentar tratamiento con reposo y AINE.
d) Seguramente no persistan lesiones residuales al curar.
e) Asimismo deben descartarse otras asociaciones
como infecciones bacterianas, enfermedades sis-
tmicas y embarazo.
180. Granjero de 40 aos acude a Urgencias por un
cuadro febril con malestar general y artromial-
gias. En la exploracin se descubre una lesin
anular eritematosa en la pierna derecha, de 8 cm
de dimetro, con centro claro. La serologa para
Borrelia burgdorferi es positiva. Cul ser el me-
jor tratamiento?
a) Penicilina intramuscular.
b) Tetraciclinas.
c) Corticoides tpicos.
d) Cotrimoxazol.
e) Lindane (hexacloruro de gammabenceno).
181. En qu fase del ciclo celular se produce la dupli-
cacin de las molculas de ADN nuclear?
a) Fase G0
b) Fase G1
c) Fase G2
d) Fase S
e) Fase M
182. Cul de los siguientes factores NO inuye en la
difusin de los gases a travs de la membrana al-
veolocapilar?
a) Gradiente de presin del gas entre ambos lados de
la membrana.
b) Concentracin de surfactante en el alveolo.
c) Espesor de la membrana.
d) Supercie de la membrana.
e) Coeciente de difusin del gas.
183. Un varn alcohlico indigente es remitido al
Servicio de Urgencias en coma. En la analtica
obtenemos los siguientes datos: pH 7,1, pO
2
99
mmHg, PCO
2
25 mmHg, creatinina 2 mg/dl, urea
50 mg/dl, sodio 140 mmol/l, potasio 4 mmol/l, bi-
carbonato 5 mmol/l, cloro 90 mmol/l, protenas 6
g/dl, glucosa 112 mg/dl, osmolalidad 320 mosm/
kg. Orina: no se observan cuerpos cetnicos, so-
dio 20 mmol/l, potasio 20 mmol/l, pH 5,5 y gluco-
suria negativa. Cul es la causa ms probable
de la acidosis?
a) Acidosis respiratoria.
b) Cetoacidosis.
c) Acidosis lctica.
d) Intoxicacin por metanol.
e) Insuciencia renal.
booksmedicos.org
-30-
SI MULACRO 1
184. La secrecin de cidos y bases orgnicas a nivel de
la nefrona tiene lugar exclusivamente en:
a) Tubo proximal.
b) Segmento descendente delgado del asa de Henle.
c) Segmento ascendente delgado del asa de Henle.
d) Segmento ascendente grueso del asa de Henle.
e) Tbulo distal.
185. El calcio es esencial para todos los siguientes pasos
en la cascada de la coagulacin EXCEPTO:
a) Activacin del factor IX por el XI.
b) Activacin del XI por el XII.
c) Activacin del X por el IX.
d) Formacin de la trombina.
e) Formacin de protrombina.
186. De los siguientes frmacos antidepresivos, seale
cul tiene un mecanismo de accin dual, sobre la
serotonina y la noradrenalina:
a) Citalopram.
b) Reboxetina.
c) Tranilcipromina.
d) Nefazodona.
e) Venlafaxina.
187. Cul de los siguientes cuadros NO considerara
de entrada entre las etiologas de un dolor en fosa
ilaca derecha?
a) Apendicitis aguda.
b) Gestacin tubrica.
c) Salpingitis.
d) Isquemia mesentrica aguda.
e) Hematoma de la vaina de los rectos.
188. Qu actitud tomara ante un paciente ingresado
por presentar un infarto de miocardio y que pre-
senta una taquicardia ventricular sostenida, aun-
que sin deterioro hemodinmico?
a) Cardioversin elctrica.
b) Lidocana (1 o 2 dosis de 50 a 100 mg).
c) Masaje cardaco.
d) Bretilio.
e) Verapamil i.v.
189. Ante un paciente en el que, tras sufrir una luxa-
cin anterior de hombro, se detecta una disminu-
cin en la fuerza para la abduccin del hombro,
debe sospecharse:
a) Desinsercin de la cabeza medial del deltoides.
b) Tendinitis del supraespinoso.
c) Fractura acromial asociada.
d) Lesin del nervio circunejo.
e) Fractura humeral asociada.
190. Slo una de las armaciones que se hacen sobre el
componente secretor es cierta. Selela:
a) Es una molcula de la superfamilia de la inmuno-
globulinas.
b) Es secretado por los linfocitos T.
c) Es secretado por los linfocitos B.
d) Su principal funcin es evitar que las IgA uyan
desde la luz del tubo intestinal o rbol traqueal
hacia la submucosa.
e) Tiene actividad antiproteasa similar a la de alfa1
antitripsina.
191. Acerca de la patologa que afecta a un nio con
antecedentes de faringitis hace dos semanas, que
presenta edema periorbitario, hematuria macros-
cpica, y en la analtica, elevacin de la VSG, de
la creatinina srica, hipoalbuminemia, y disminu-
cin de los niveles de complemento, seale lo que
le parece INCORRECTO:
a) Si la infeccin previa hubiese sido cutnea, proba-
blemente no se encontrara respuesta inmunitaria
frente a la estreptolisina (ASLO).
b) El complemento se normaliza a las 12-16 semanas.
c) Los cultivos de faringe y la determinacin de anti-
cuerpos contra el agente infeccioso pueden perder
valor diagnstico si se instaura terapia antimicro-
biana precozmente.
d) Una proteinuria en rango nefrtico es infrecuente
en estos casos, pero no excluye el diagnstico.
e) Normalmente, la concentracin de Na+en orina
es baja.
192. Pueden ser procesos linfoproliferativos T todos los
siguientes, EXCEPTO:
a) Leucemia linfoide crnica.
b) Macroglobulinemia de Waldenstrm.
c) Micosis fungoide.
booksmedicos.org
-31-
SI MULACRO 1
d) Sndrome de Szary.
e) Tricoleucemia.
193. En una prueba de signicacin estadstica, cun-
do se incurre en un error tipo II o beta?
a) Al concluir que una diferencia existe cuando en
realidad no existe.
b) Al rechazar la hiptesis nula cuando es cierta.
c) Al no rechazar la hiptesis nula cuando es cierta.
d) Al concluir que una diferencia no existe cuando
en realidad existe.
e) Al aceptar la hiptesis alternativa cuando es falsa.
194. La Leishmania donovani tiene histotropismo por:
a) Leucocitos.
b) Neuronas corticales.
c) Sistema reticuloendotelial.
d) Miocitos.
e) Clulas del tbulo contorneado distal.
195. Sealar cul de estas armaciones NO es cierta:
a) El nervio facial en el odo medio va por el conduc-
to de Falopio.
b) El nervio facial entra en el odo por el CAI.
c) El nervio facial sale del temporal por el agujero
estilomastoideo.
d) El nervio facial tiene bras motoras sensitivas y
parasimpticas.
e) La cuerda del tmpano no es una rama del facial.
196. En relacin con el virus de la hepatitis tipo B, indi-
que cul de las siguientes le parece INCORRECTA:
a) Est claramente establecido el carcter citoptico
directo del virus de la hepatitis B.
b) El virus de la hepatitis de tipo B se ha identicado
a nivel extraheptico.
c) Existen dos variantes con importancia, medite-
rrnea y mutante de escape.
d) El HBeAg y las protenas pre-S1 y pre-S2 se ex-
presan durante los perodos de replicacin mxi-
ma.
e) En el 1 al 5 % de las infecciones agudas no se
detecta HBsAg por tener concentraciones dema-
siado bajas.
197. La longitud del intestino grueso es de:
a) 6 a 7 pies.
b) 120 a 160 cm.
c) 120 a 160 pies.
d) 12 a 16 cm.
e) 6-7 metros.
198. La arteria pedia es continuacin de la arteria:
a) Tibial anterior.
b) Tibiar posterior.
c) Popltea.
d) Peronea.
e) Arteria femoral profunda
199. El hgado obtiene su energa principalmente a
partir de:
a) Glucosa
b) cidos grasos
c) Glicerol
d) Aminocidos
e) Fosfolpidos
200. La unidad funcional del rin es:
a) Macula densa
b) Nefrona
c) Aparato yuxtaglomarular
d) Clices mayores
e) Mesangio
booksmedicos.org
booksmedicos.org
176
177
178
179
180
181
182
183
184
185
186
187
188
189
190
191
192
193
194
195
196
197
198
199
200
141
142
143
144
145
146
147
148
149
150
151
152
153
154
155
156
157
158
159
160
161
162
163
164
165
166
167
168
169
170
171
172
173
174
175
106
107
108
109
110
111
112
113
114
115
116
117
118
119
120
121
122
123
124
125
126
127
128
129
130
131
132
133
134
135
136
137
138
139
140
71
72
73
74
75
76
77
78
79
80
81
82
83
84
85
86
87
88
89
90
91
92
93
94
95
96
97
98
99
100
101
102
103
104
105
36
37
38
39
40
41
42
43
44
45
46
47
48
49
50
51
52
53
54
55
56
57
58
59
60
61
62
63
64
65
66
67
68
69
70
1
2
3
4
5
6
7
8
9
10
11
12
13
14
15
16
17
18
19
20
21
22
23
24
25
26
27
28
29
30
31
32
33
34
35
NUMERO DE MESA ...................................................................................................
VERSION DEL CUESTIONARIO DE EXAMEN ................................................................
N DE EXPEDIENTE ......................................................................................................
N DE D.N.I. O EQUIVALENTE PARA EXTRANJEROS ...................................................
APELLIDOS Y NOMBRE ................................................................................................
ADVERTENCIAS:
a) Escriba con BOLIGRAFO sobre superficie dura y lisa.
b) No utilice lpiz, rotulador o pluma de tinta que se pueda borrar.
c) Indique la respuesta que crea correcta en forma claramente legible.
d) Las respuestas ilegibles o confusas o las indicadas con otra clase diferente de signos se penalizarn como incorrectas.
e) Las equivocaciones deben subsanarse tachando ntegramente la respuesta errnea, sin que sta quede visible, y escribiendo al
lado la respuesta elegida.
f) Si inutilizara esta hoja de respuesta devuelva los tres ejemplares de que se compone a la Mesa de Examen para recibir otra de
repuesto y no olvide consignar sus datos personales.
C
D
C
A
C
D
E
D
D
B
C
C
E
B
D
C
C
E
D
C
A
B
B
C
A
D
C
A
B
E
B
C
E
D
B
B
B
E
C
E
A
C
D
D
C
D
E
D
B
B
C
C
E
D
A
D
D
C
D
C
D
C
E
D
B
A
C
B
D
A
E
E
D
E
C
C
D
B
D
C
C
D
D
C
C
C
B
E
B
C
D
B
E
D
B
C
D
D
A
D
E
B
D
D
D
D
D
B
E
E
A
E
B
E
C
D
C
D
E
B
E
B
A
E
D
D
B
C
C
A
C
B
C
D
B
B
E
C
D
E
D
B
B
A
B
A
B
A
D
E
C
A
C
C
D
C
B
D
E
A
C
C
C
B
B
B
D
E
D
A
B
E
A
C
B
E
C
A
A
B
D
B
D
A
E
E
D
B
D
A
B
B
D
C
E
A
B
A
A
B
respuestas simulacro 1 Plantilla
booksmedicos.org

Anda mungkin juga menyukai